Rheum Qs

¡Supera tus tareas y exámenes ahora con Quizwiz!

A 34-year-old male, with an 8-month history of arthralgias, is referred by his primary care physician because his laboratory studies were positive for RF and anti-CCP antibodies. The anti-CCP antibody is associated with which of the following? A) Early development of bone erosions B) The development of a lupus nephritis C) The development of amyloidosis of the kidneys D) Early development of rheumatoid nodules E) The development of a rheumatoid arthritis/lupus overlap syndrome

Answer A. Early development of bone erosions Explanation The presence of anti-CCP antibody is associated with the development of more severe disease as defined by the presence of early bone erosions. Although, rheumatoid nodules are associated with severe disease, no association exists with the anti-CCP antibody. Similarly, no association exists between anti-CCP and the remaining choices.

with retinal vasculitis with the venous system being more involved. A diagnosis of Behçet's had been considered because of a previous presentation with iritis several years prior to this. She also has a lengthy history of oral ulcers and had been diagnosed with complex oral aphthosis by a dermatologist several years ago. Two months ago, she presented to her gynecologist with severe labial ulcerations. These have continued to be a problem over the past few months. She is not short of breath or coughing but is quite concerned about a new onset of an acne-like rash on her face and trunk. The joints of her fingers have become very painful. She underwent a subtotal thyroidectomy 18 months ago and is presently on thyroid replacement. On exam, she is well nourished and well hydrated with T 97.8° F, BP 112/84. Cardiac and respiratory exam is benign. Oral mucosa reveals multiple aphthae. Genital exam revealed a 2 x 2 cm healing ulcer over the left labial area. Cutaneous exam is positive for a hyperpigmented acne-like rash over the face and chest. Investigations reveal a Sed rate of 37, RF and ANA negative, hemoglobin of 11.2, and WBC of 9.3. Which of the following drugs would not be reasonable options for treating her condition? A) Thalidomide B) Hydroxychloroquine C) Azathioprine D) Colchicine E) Cyclophosphamide

Answer B. Hydroxychloroquine Explanation Her diagnosis is compatible with Behçet syndrome. Retinal vasculitis with venous involvement is quite typical of Behçet's. The iritis is also suggestive. Acne plus arthralgias plus recurrent oral and genital ulcers are typical of this syndrome. All of the options except hydroxychloroquine can be used to treat Behcet's. Colchicine is used to treat apthous ulcers and arthritis. Thalidomide is reserved for more serious mucous membrane involvement given its significant side effect profile. It also requires absolute contraception and regular nerve conduction velocity testing. Azathioprine and cyclophosphamide can be used in more severe disease (i.e., sight-threatening eye involvement). Hydroxchloroquine is not indicated for treatment of Behçet syndrome.

A 57-year-old female presents to your office with numbness and tingling of the right wrist and hand. This has been present for the past six weeks and seems to be somewhat worse at night. These symptoms are improved by flicking her wrist. She has no symptoms on the other side. Her general medical inquiry is entirely negative. She has no paresthesias in the lower extremities. She has never had DM or HTN and is not on any medications whatsoever. She is married, is a housewife, and has two children. She has been typing more on her computer recently. On exam she is alert and oriented, well nourished, and well hydrated. BP 112/72, P 72. Cardiorespiratory exam is negative. MSK exam is negative. Neurological exam is negative, completely without any sensory or motor deficits. Which of the following is the most likely diagnosis? A) Median nerve entrapment B) Cervical disc degenerative disease C) Brachial plexopathy D) Ulnar neuropathy

A. Median nerve entrapment Explanation The entrapment is presumably at the wrist. The positive flick test is very suggestive. Tinel's and Phalen's signs are fairly insensitive and nonspecific. She has none of the typical risk factors, namely DM, hypothyroidism, pregnancy, oral contraceptive use, or an inflammatory arthritis. Carpal tunnel entrapment of the idiopathic variety is still the most common cause. At this stage, there are many treatment options: A resting wrist splint, an injection into the canal, NSAID therapy, or simply conservative measures to include less typing.

A 42-year-old gentleman presents with severe headache and fever. He is seen in his local emergency department and is complaining of a slightly stiff neck. Meningitis is considered, and a CSF tap is completed, which is negative. CSF protein is normal. No white cells were found in the CSF. As part of the workup, a CT scan is completed in the emergency department and significant sphenoidal sinus enlargement and engorgement is noted. He has a history of a 15-pound weight loss over a 6-week period prior to his admission. He has had significant headaches. In general, he has been in excellent health. He has never had sinusitis before, though he does remember having otitis media diagnosed by his primary care physician about 8 weeks ago, treated with oral amoxicillin for 7 days. He has also noted a rash over the anterior shins, which is non-itchy. This developed over the last 2 days. Over the past 24 hours, there has been some numbness and tingling of both lower extremities. On exam, he is oriented but a little apathetic. T 102.3° F, BP 112/72, P 94. Neck is supple. Cardiac exam is negative and lungs reveal some coarse wheezing. The abdomen is soft with hepatosplenomegaly. Joint exam is negative. Cutaneous exam reveals a fine, erythematous, palpable purpuric lower extremity rash. Neurologic exam reveals no sensory or motor deficits. Investigations reveal a normal CXR. His hemoglobin is 9.8, WBC is 15.3, and platelet count is 276. His sed rate is 123, creatinine is 1.2, and U/A reveals 2+ RBCs with red cell casts. Which of the following is most likely to provide a diagnosis? A) c-ANCA (anti-proteinase 3) B) Anti-SCL70 C) Anti-dsDNA antibodies D) ANA E) p-ANCA

A. c-ANCA (anti-proteinase 3) Explanation He has sinusitis, vasculitis, possible early peripheral neuropathy, and he is quite ill. The diagnosis of granulomatosis with polyangiitis (formerly Wegener's) is quite likely. c-ANCA (plus anti-PR3) is highly suggestive of the diagnosis and often circumvents the need for biopsy. The sphenoidal sinus obviously would not be easily accessible to an ENT surgeon. However, if ANCA serologies were negative and if some other site was more accessible, such as an axillary sinus, then biopsy may prove useful.

A 47-year-old female presents with symptoms of "grittiness" of her eyes and dryness of her mouth. She has had problems reading recently. She was recently seen by an ophthalmologist, who thought she might have early Sjögren's syndrome, and a Schirmer's test was apparently positive. Other than occasional pain in the wrists, she really has no joint problems. She denies rashes, SOB, cough, dysphagia or dysuria. She recently underwent a benign breast biopsy. Her family history is noncontributory. On examination today, she is alert and oriented. Height 5'5", weight 112, P 76. She is not pale or jaundiced. The cardiorespiratory exam is benign. The abdomen has no masses. There is no lymphadenopathy. Complete musculoskeletal exam is negative. Cutaneous exam is benign. Exam of the oral mucosa reveals diminished salivary pool. Special investigations reveal her CBC to be normal. CMP is normal. ANA is strongly positive and the anti-SSA is positive. This patient is not at risk for which of the following? A) Barrett esophagus B) Non-Hodgkin lymphoma C) Renal tubular acidosis D) Peripheral neuropathy E) Lymphocytic interstitial pneumonitis

Answer A. Barrett esophagus Explanation Patients with Sjögren's develop all kinds of extra-glandular involvement. Esophageal involvement is more typical of scleroderma.

A 35-year-old African-American woman diagnosed with diffuse scleroderma 2 years previously and followed by an outside provider presents to your emergency department complaining of severe headaches and flushing for 3 days. She was in her usual state of decent health until 3 days ago, when she awoke with a "pulsing" headache that has not resolved. The pain has reduced in intensity but episodically returns to a very intense pain. This morning she developed a nosebleed, which has never happened before. Review of systems is negative for nausea, vomiting, diarrhea, fevers, chills, and weight loss. She reports normal urinary frequency without flank pain or dysuria. PE: BP 210/185, HR 98 bpm, RR 14, Afebrile PE significant for obvious thickening of the skin on the face and sclerodactyly Funduscopic exam shows normal retinas and no papilledema. Non-displaced PMI, normal cardiac exam Normal lung exam No ulcerations on the fingers No peripheral edema LABORATORY RESULTS: CBC: Normal Na: 135 mEq/L K: 3.2 mEq/L Cl: 103 mEq/L HCO3: 23 mEq/L BUN: 32 mg/dL Creatinine: 4.3 mg/dL Creatinine 3 months ago: 1.2 mg/dL Albumin: 3.7 g/dL LFTs: Normal Troponins: < 0.01 ng/mL at 2 measurements 2 hours apart Urinalysis: SG 1.025, pH 6.0, protein 1+, blood none, glucose none, WBC 3-5, RBC 0 Electrocardiogram: Sinus rhythm with ST depressions 1.2 mm in leads 1, aVL, and V5-V6 consistent with LV strain. No hypertrophy is indicated. Which of the following is the most appropriate pharmacologic management? A) Captopril 12.5 mg PO now B) Atenolol 50 mg PO now C) Hydralazine 50 mg PO and isosorbide dinitrate 10 mg PO now D) Extended-release nifedipine 90 mg PO now E) HCTZ 25 mg PO now

Answer A. Captopril 12.5 mg PO now Explanation This is a case of scleroderma renal crisis with malignant hypertension. In diffuse scleroderma, renal crisis is one of the most serious early complications. This is usually seen in young women with early rapid progression of this disease. African-American patients with diffuse disease have a much higher incidence of renal crisis compared to Caucasian counterparts (21% compared to 7%). Differential diagnosis for this presentation in a patient without obvious scleroderma would include HUS, TTP, antiphospholipid antibody syndrome, and malignant nephrosclerosis in those with preexisting hypertension. But in settings where scleroderma is a known diagnosis, the diagnosis is straightforward. Screening for this complication in diffuse scleroderma patients is helpful in preventing renal failure; blood pressure should be monitored monthly and renal function monitored every 3 months. ACE inhibitors can prevent this complication and treat renal crisis, even in the setting of acute renal failure. ACEIs also appear to reduce mortality. This complication was seen in up to 25% of patients in the past, but much less often now due to the use of ACEIs. Calcium channel blockers are typically added when ACEIs fail to control the blood pressure adequately. Beta-blockers and diuretics are not known to exert any affect on reversing the renal disease in these patients. Isosorbide and hydralazine in combination are used to treat African-American patients with heart failure, not for scleroderma renal crisis. Malignant hypertension, plus acute renal failure, microangiopathic hemolytic anemia, and retinal hemorrhages would be the expected findings with scleroderma. Again, ACEIs, not beta-blockers, are the drugs of choice to prevent and treat renal crisis.

A 36-year-old man with a history of gout returns for follow-up. He has had a 2-day history of cough and fevers today. Chest x-ray shows a RLL infiltrate. PMH: CRI baseline Cr 2.0. Meds: allopurinol 200 mg a day, colchicine 0.6 mg a day, citalopram 20 mg a day Which drug would be most dangerous to prescribe? A) Clarithromycin B) Levofloxacin C)Chloramphenicol D) Azithromycin E) Erythromycin

Answer A. Clarithromycin Explanation Always think of clarithromycin as the king of drug interactions. It has a major effect on P450 drug metabolism and can markedly increase levels of other coadministered drugs. This is especially true of colchicine, where the majority of deaths due to colchicine drug interactions in the past have been due to the clarithromycin drug interaction.

A 57-year-old grandmother presents with 3 months of difficulty lifting her 2-year-old grandchild, difficulty initiating her swallow, and dyspnea with minimal exertion. Her laboratory evaluation revealed a CPK of 4,500. Her muscle biopsy reveals an inflammatory infiltrate comprised of lymphocytes and macrophages with muscle fiber necrosis and regeneration (consistent with polymyositis). The additional evaluations of this patient should include which of the following? A) Colonoscopy, Pap smear, mammogram, and chest radiograph B) Chest, abdomen, and pelvis CT scan C) Cervical, thoracic, and lumbosacral spine MRI D) ANA, rheumatoid factor, ANCA, and angiotensin converting enzyme levels E) HIV, CMV, and herpes antibody levels; a hepatitis panel

Answer A. Colonoscopy, Pap smear, mammogram, and chest radiograph Explanation The question refers to a patient with polymyositis. In newly diagnosed PM/DM patients older than 50 years, the muscle disease is possibly a paraneoplastic syndrome. PM/DM in these older patients has been associated with multiple malignancies, both lymphoproliferative and solid. As a result, these patients must be evaluated for cancer. Several papers have indicated that the most cost-effective cancer evaluation is routine health maintenance studies; a chest radiograph; and anything else suggested by the patient's history, physical examination, or laboratory data.

A 30-year-old woman with carpal tunnel syndrome comes to you after having been treated by another physician in town with nonsteroidal antiinflammatories for the past few months without much improvement. She has also been using "splints" regularly without much improvement. She has no loss of motor function. EMG and nerve conduction studies are consistent with delayed conduction in the distribution of the median nerve. Before sending her to surgery, which one of the following should you try? A) Corticosteroid phonopheresis into the carpal tunnel. B) Repeat the EMG and nerve conduction studies. C) Treatment with a tricyclic antidepressant at bedtime. D) Give 2 weeks of broad-spectrum antibiotics.

Answer A. Corticosteroid phonopheresis into the carpal tunnel. Explanation Therapeutic ultrasound using phonopheresis with steroids or NSAIDs have been found to be effective in managing patients with carpal tunnel syndrome. The other choices are not indicated. Repeating the EMG/NCS would not help at this point.

A 43-year-old heart transplant recipient comes to the hospital for evaluation of severe foot pain. This pain has been present for 12 hours and hurts even with the slightest pressure to the foot. He received his transplant 3 months ago and has had no problems with function or rejection. He has no prior musculoskeletal problems, fever, or other new symptoms. PMH: Hx of duodenal ulcer disease and transplant. Meds: nizatidine, prednisone, cyclosporine, amlodipine, and aspirin. Labs: WBC: 13,000, Hct: 40, SGOT: 30, SGPT: 22. Of the following, what is the most likely cause of this patient's problem? A) Cyclosporine B) Alcohol use C) Nizatidine D) Group A Streptococcus E) Aspirin

Answer A. Cyclosporine Explanation This patient has typical clinical features of gout: acute onset of severe foot pain. He is on cyclosporine, a drug that increases uric acid levels and frequently can cause gout. Although aspirin in low doses can slightly increase gout risk, cyclosporine is much more likely (up to 30% develop gout). Alcohol also can increase the risk of gout, but no history of alcohol use is given in this case.

Which of the following therapies for rheumatoid arthritis is associated with retinopathy? A) Gold B) Hydroxychloroquine C) Penicillamine D) Methotrexate E) Prednisone

Answer B. Hydroxychloroquine Explanation Patients on hydroxychloroquine require a baseline ophthalmologic examination and periodic examinations thereafter. Methotrexate, gold, and penicillamine are associated with leukopenia, renal toxicity, and pneumonitis. Prednisone can cause osteopenia and osteonecrosis.

A 24-year-old female presents with 6 months of symmetric MCP, PIP, and wrist joint synovitis. Her ANA and RF are positive, and her hand radiographs do not indicate erosions. Which of the following features suggests systemic lupus erythematosus rather than rheumatoid arthritis? A) Her additional history of 3 miscarriages B) Her physical examination finding of synovitis C) Her additional radiographic finding of bilateral pleural effusions D) Her additional history of sicca symptoms E) Her additional laboratory finding of neutropenia

Answer A. Her additional history of 3 miscarriages Explanation Systemic lupus erythematosus (SLE) and rheumatoid arthritis (RA) have many similarities including autoantibodies, sicca symptoms, serositis (causing pleural effusions,) and synovitis. However, RA causes bone erosions and in the Felty syndrome subset, patients can develop antineutrophil antibodies and neutropenia. While SLE can cause leucopenia, it does not typically cause neutropenia. However, a lupus patient with the antiphospholipid antibody can experience multiple miscarriages, an occurrence that does not occur in RA.

A 47-year-old woman with a 5-year history of reflux and an 18-month history of Raynaud's is seen by her primary care physician with a history of worsening of her reflux and of solids occasionally getting "stuck in her throat." She particularly has problems swallowing pills and more recently has had to cut her food up into much finer pieces in order to get it down, and is using more liquids to do so. She has no joint pain but is concerned that she is losing grip in her fingers. She is unable to crochet the way that she would like to because of loss of dexterity. She is not short of breath. She has had no other major problems. She denies any joint pain. She is taking a proton pump inhibitor twice daily. She is well nourished. Her blood pressure is 137/73 and pulse is 76. Cardiac exam is benign. Respiratory tract is without crackles. Her joints are normal. Cutaneous exam reveals mild sclerodactyly with positive vital nailfold capillaroscopy. She has some telangiectasias on her chest. Lab workup reveals hemoglobin of 11.7, WBC of 4.9, and normal platelet count. Creatinine is 0.9. ANA is positive, and anti-centromere antibodies were strongly positive. U/A is clear. Which of the following investigations would be appropriate as part of her long-term follow-up? A) Occasional ECHO to estimate pulmonary pressure B) Constant monitoring of ANAs C) Small bowel follow-through D) Serum protein electrophoresis

Answer A. Occasional ECHO to estimate pulmonary pressure Explanation This patient has CREST syndrome, a limited cutaneous form of scleroderma. While the prognosis is generally regarded as one of the more benign variants of scleroderma, nevertheless, pulmonary HTN is well described. Therefore, it would be important to look for the early presence of pulmonary HTN, particularly given the newer groups of medications that could potentially arrest this. A small bowel follow-through would be indicated for signs of pseudo-obstruction of the small bowel, which this patient does not have. ANA titers do not correlate with disease activity. An SPEP is not indicated for scleroderma patients.

A 61-year-old woman with a hypothyroidism presents with 3 months of abdominal pain. The patient's pain is diffuse, worse after meals, is associated with nausea but no vomiting, and has been progressively worsening over the last 2 months. She was seen in an urgent care facility one month ago, diagnosed with H. pylori by serology, and treated with triple therapy for H. pylori gastritis. The patient has lost 20 pounds during this time. The pain did not improve with treatment, and she comes in to clinic for follow-up. A review of systems is significant for diffuse myalgias during this time. Fatigue and an episode of foot drop that occurred two months ago has since resolved. She has no diarrhea, melena, or hematochezia. Her past medical history is otherwise unremarkable; she is up-to-date with her routine health maintenance. She does not smoke or drink alcohol and is not sexually active. Physical exam is significant for a blood pressure of 178/94, heart rate of 94, and normal oxygen saturation. The patient appears chronically ill; her heart and lung examination is normal. She has diffuse abdominal pain without rebound or masses. Her examination is otherwise normal. Laboratory studies reveal a normal WBC, hemoglobin of 10.8, MCV of 82, normal platelets. Her ESR is 94. Urinalysis is significant for large blood, negative leukocyte esterase, and trace ketones. Urine microscopy shows 352 red blood cells/hpf, with 20 white cells/hpf. What is the most likely cause of the patient's symptoms? A) Polyarteritis nodosa B) Resistant Helicobacter pylori infection C) Colon cancer D) Crohn disease E) Nephrolithiasis

Answer A. Polyarteritis nodosa Explanation The combination of postprandial abdominal pain, new onset hypertension, hematuria, and elevated inflammatory markers is highly suggestive of polyarteritis nodosa (PAN), a small and medium vessel vasculitis. PAN typically affects the kidneys and spares the lungs, unlike other ANCA-positive vasculitides. Neither resistant H. pylori infection nor nephrolithiasis would be associated with weight loss or increased inflammatory markers. Crohn disease could present in a similar way; however, the lack of melena, hematochezia, or oral ulcers and the recent onset of symptoms make this diagnosis unlikely.

A 31-year-old physician's assistant is seen for worsening Raynaud's. She has had classic triphasic color changes in her fingers for the past 10 years. She has never had ulcers on her fingers. She has also had a history of gastroesophageal reflux. The only other medical history of note is that she was diagnosed with anorexia nervosa 10 years ago but feels that she is now currently "healed." No joint pain and no rashes of note. No problems swallowing food. Examination reveals a healthy 5' 2" female whose weight is 104 lbs. She is well nourished and well hydrated. Skin is normal without sclerodactyly, telangiectasia, or pulp atrophy. However, 2 of the digits on her right hand reveal an abnormal vital nailfold capillaroscopy with dilated capillaries. The remainder of her exam is benign. Which of the following serologies is not compatible with a diagnosis of primary Raynaud's? A) Positive anti-centromere antibodies B) Rheumatoid factor C) Anti-nuclear antibody D) Normal ESR

Answer A. Positive anti-centromere antibodies Explanation She has had stable Raynaud's for 10 years. Therefore, to suddenly now develop a connective tissue disease process is unlikely. The dilated capillaries are of concern, but not present in all of her digits. ANA's are found in 20% of patients with primary Raynaud's. Anti-centromere is seen in CREST. Rheumatoid factor is 85% specific for rheumatoid arthritis, but it can also be false-positive for other conditions. ESR would be normal with primary Raynaud disease.

A frustrated 76-year-old man is seen as an outpatient. He has a 20-year history of joint pain. Initially, he began with some stiffness and swelling of the left knee, which rapidly spread to his right knee. He was told that he had OA and had several unsuccessful corticosteroid injections into both knees. His orthopedist was not inclined to replace the knee joints at that time, given his age. Over the years, the knees slowly began to bother him less but certainly made ambulation uncomfortable. About 5 years ago, he began to develop right groin pain and was told that he had severe "arthritis" in the right hip area. Hip replacement was suggested and, because of ongoing limitation of mobility, this was successfully completed 2½ years ago. More recently, he has begun to experience some pain in the base of his right thumb. He has tried acetaminophen and numerous OTC NSAIDs and had been placed on nabumetone, sulindac, and diclofenac. He developed significant abdominal pain while on the nabumetone and subsequently had an endoscopy and was told that he had a small gastric ulcer. He was told not to take antiinflammatories any longer. More recently, his knees have been bothering him significantly. The rest of the history is fairly benign. He has moderate HTN and is on an ACE inhibitor. On examination, he is alert, oriented, and well nourished. BP is 137/91 and pulse is 76. Cardiorespiratory exam is benign. Complete musculoskeletal exam reveals bilateral asymmetric Heberden nodes with some squaring of both thumbs. The elbows and shoulders have a good range of motion. Both hips have reasonable internal and external rotation, but he has fairly significant crepitus of both knees. He has fairly obvious varus deformities with some laxity of the lateral collateral ligaments on both sides. He has marked hallux valgus of the right toe. Cutaneous exam is benign. Which of the following is not a reasonable therapeutic option? A) Prednisone 10 mg qd B) Intraarticular glucocorticoid C) Celecoxib D) NSAID + proton pump inhibitor

Answer A. Prednisone 10 mg qd Explanation This patient has osteoarthritis, which can be progressive in some patients. The "incorrect" therapeutic option would be prednisone 10 mg qd. This patient is elderly and has had a previous history of peptic ulcer and, therefore, is classically at risk for life-threatening GI bleed. The COX-2 inhibitors have been developed specifically for this purpose, and there are outcome studies to demonstrate a significant reduction in life-threatening GI bleeds with this class of drugs. However, their cardiovascular effects have resulted in many being removed from the market with only celecoxib remaining. Because of this, many recommend NSAIDs with proton pump inhibitors instead, although NSAIDs have recently been shown to increase cardiovascular risk as well, with the exception of naprosyn. Hydrocodone is obviously addictive and is the last option for this patient. There is no reason to give him systemic corticosteroid therapy.

A 30-year-old African-American woman presents with new onset of serositis. She has been well except for a cardiac arrhythmia, which required her to be started on a cardiovascular drug a few months ago. She is noted on exam to have serositis, but she has no evidence of renal or CNS involvement. She does not have a rash. Which of the following is true about her condition? A) Procainamide could have induced this. Anti-histone antibodies are rarely positive. B) Rash is usually more common than serositis with this presentation. C) The absence of renal involvement in this case is unusual. D) Fever is uncommon.

Answer A. Procainamide could have induced this. Explanation She likely has drug induced Lupus-like syndrome. Procainamide classically is described doing this, and anti-histone antibodies are usually positive. Rash is rare and serositis is the most common presentation. Fever is common as well. CNS and renal involvement are rare.

Sausage digits with the additional finding of pitted nail beds are associated with which of the following inflammatory arthritides? A) Psoriatic arthritis B) Rheumatoid arthritis C) Reactive spondyloarthropathy D) Ankylosing spondylitis E) Sjögren arthritis

Answer A. Psoriatic arthritis Explanation Psoriatic arthritis is associated with nail pitting and sausage digits. Ankylosing spondylitis and reactive spondyloarthropathy are definitely associated with enthesopathies (causing sausage digits), but only psoriasis is associated with both the nail changes and the sausage digits.

A 47-year-old woman presents to her internist with painful swelling of her joints. Specifically, she has had problems with her finger joints, wrists, elbows, shoulders, knees, and feet, and complains of early morning stiffness lasting several hours. She is very concerned because her grandmother suffered from very bad RA. Her older sister has been diagnosed with apparent lupus. Her medical history is benign except for a history of HTN, which has been controlled with diuretics for several years. Her menses have become a little irregular of late. She is sleeping poorly and states that her energy levels have been almost "non-existent." She admits to significant stress, presently undergoing a divorce. Exam reveals a healthy-appearing woman with BP 147/92, P 92, R 22, weight 212, height 5'6". She is not pale. Cardiorespiratory exam is benign. Abdomen is obese but no masses are appreciated. A complete MSK exam reveals no synovitis about the PIP or MCP joints. Grip strength is excellent. The wrists, elbows, and shoulders all have a good ROM and no synovitis is detected. No effusions are noted in the knees and no crepitus. No metatarsalgia or clawing of the feet. Cutaneous exam is without psoriasis. She is tender on the lateral aspects of the shoulders and over the subtrochanteric areas bilaterally. She is tender in the lower lumbar areas and over the left anterior chest wall. As part of her workup, she is found to have a positive RF and ANA, and has a Sed rate of 9. Her hemoglobin is 12.7, WBC is 7.6, and platelet count is 212. Which of the following statements is true? A) She has an overlap of RA and SLE. B) She probably has fibromyalgia but should be monitored for the remote possibility of early RA. C) She has RA and should begin methotrexate immediately. D) She probably has SLE, and anti-dsDNA antibodies should be checked.

Answer B. She probably has fibromyalgia but should be monitored for the remote possibility of early RA. Explanation The positive RF and ANA probably relate to her family history. Her exam is negative for synovitis. Given the tender points, her stress level, poor sleep pattern, and malaise, the diagnosis of fibromyalgia seems most likely. Given her mother's history of RA, she obviously has an appropriate fear of this condition. She should be reassured about the absence of synovitis and should be closely monitored. An x-ray of the hands and feet would be reasonable to exclude the presence of any erosive changes. She should be advised to come back in should the joints become "swollen."

A 44-year-old Caucasian male presents to the emergency department with a swollen left knee for 4 days. He has experienced 2 episodes of podagra within a year. He denies any other medical problems, but admits to drinking beer on weekends. He does not take any medications. Physical exam: Temp: 99.6° F, BP 140/86, P 90 reg, RR 12 Skin: Mild erythema and warmth over dorsum of left knee Lungs: CTA Cor: N1 S1, S2; no murmurs, rubs, or gallops Abd: Soft, with normal bowel sounds MS: No tophi or nodules noted; large left knee effusion is present Knee arthrocentesis removed 20 cc of yellow, turbid fluid. Aspiration confirmed negatively birefringent monosodium urate crystals. Which of the following strategies will be most effective in preventing further episodes of joint attacks? A) Start allopurinol and titrate to serum uric acid < 6.0 mg/dL. B) Dietary and lifestyle changes. C) Start ibuprofen 800 mg PO every 6 hours. D) Alkalinize the urine. E) Start IV colchicine as prophylaxis for 6 months.

Answer A. Start allopurinol and titrate to serum uric acid < 6.0 mg/dL. Explanation Per the American College of Rheumatology 2012 gout management guidelines, all patients with gout should be counseled about dietary and lifestyle changes, but this is difficult for most patients to accomplish. However, urate lowering therapy (ULT) is most effective in the management of gout and should be initiated in those who have 2 or more gout attacks, with goal uric acid < 6.0 mg/dL in patients without tophi; for those with tophi, setting the goal uric acid < 5.0 mg/dL will accelerate the dissolution of tophi. Antiinflammatory prophylaxis with low-dose NSAIDs or oral colchicine is indicated when initiating urate lowering therapy. Duration of prophylaxis should be 6 months or 3 months after goal uric acid is achieved if there is no history of tophi. There is no history of nephrolithiasis in this patient for alkalinization of urine.

A 42-year-old Caucasian woman presents to your office with the chief complaint "I may have osteoporosis. Can you test me?" On further questioning, you learn that she is a very active woman and works out at a gym 3 times a week doing strength training as well as aerobic exercise. She has never smoked and rarely drinks alcohol. She is still having menses. Physical examination is unremarkable. She is 5' 10" and weighs 140 lbs. Based on your findings, which of the following is the best thing to do concerning her "osteoporosis"? A) Tell her that her risk of osteoporosis is very low, and no testing is required at this time. B) Start her on estrogen/progesterone. C) Tell her that she is at risk for osteoporosis, and you will send her to get a DXA scan to assess bone mineral density. D) Tell her you are worried that she may have a systemic disease, and you order lumbar spine films, CBC, thyroid function tests, and urinalysis. E) Tell her that her case requires referral to a specialist.

Answer A. Tell her that her risk of osteoporosis is very low, and no testing is required at this time. Explanation This patient is at low risk for fracture, and no further testing is required. Current guidelines for bone density screening indicate that the following individuals be tested: Women age 65 and older and men age 70 and older, regardless of clinical risk factors Younger postmenopausal women and men age 50-69 about whom you have concern based on their clinical risk factor profile Women in the menopausal transition if there is a specific risk factor associated with increased fracture risk such as low body weight, prior low-trauma fracture, or high-risk medication Adults who have a fracture after age 50, adults with a condition (e.g., rheumatoid arthritis) or taking a medication (e.g., glucocorticoids in a daily dose ≥ 5 mg prednisone or equivalent for ≥ 3 months) associated with low bone mass or bone loss Anyone being considered for pharmacologic therapy for osteoporosis Anyone being treated for osteoporosis—to monitor treatment effect Anyone not receiving therapy in whom evidence of bone loss would lead to treatment

A 69-year-old female with known HTN returns for follow-up. She has been previously non-compliant with her antihypertensive medication, citing financial constraints. She is meant to be on a combination of a thiazide and an ACE inhibitor, of which she has taken neither for the past 2 months. She has noticed occasional palpitations but no chest pain. Since her last visit 5 months ago, she has noticed pain in her feet. One of her sons has gout, and she is concerned that this is her problem. She wants to have her uric acid level checked today. She has some pain over the DIP joints, which has been present for several years. The right knee continues to be painful after walking 1 city block or so. She has had several episodes in which her feet have become swollen and is quite concerned about pain around the right ankle area. She denies the use of alcohol. She has noticed some polyuria and polydipsia. On exam, she is alert, well nourished, and well hydrated. BP 187/99, P 100, R 20. JVD is not elevated. Cardiac exam reveals an S4, and the lungs are clear. Complete review of MSK exam reveals mild asymmetric Heberden and Bouchard nodes. There is crepitus of both knees noted. There is bilateral 4+ lower extremity edema noted. No tophi are noted. Her labs from that date reveal a WBC of 7.3, hemoglobin of 16.2, creatinine of 1.9, and uric acid level is 8.2 (upper limit of normal in that lab 8.0). Which of the following is appropriate? A) The patient is at risk for allopurinol hypersensitivity syndrome. B) Compliance with her hypertensive medication should be discouraged. C) The patient should be initiated on allopurinol 450 mg qd. D) The drug of choice would be probenecid.

Answer A. The patient is at risk for allopurinol hypersensitivity syndrome. Explanation She is elderly, hypertensive, has renal dysfunction, and does not have crystal-proven gout. The allopurinol hypersensitivity syndrome often incorporates dramatic worsening of renal function, liver enzyme elevation, and rashes, to include Stevens-Johnson syndrome. Probenecid is contraindicated given the renal dysfunction. Allopurinol should not be initiated at such a high dose and should be initiated only after the diagnosis of gout has been proven. Recent guidelines recommend that the starting dose of allopurinol not exceed 100mg /day and should be lower in patients with moderate to severe kidney disease. Obviously, her hypertension should be well controlled

A 62-year-old postmenopausal female with a history of hypothyroidism (for which she takes levothyroxine), severe GERD, and a recent radial head fracture is diagnosed with osteopenia (based on a T-score of -2.1). In addition to prescribing calcium and vitamin D, which of the following treatment statements is true? A) This patient should receive yearly zoledronic acid. B) This patient is an ideal candidate for calcitonin. C) This patient should decrease her levothyroxine dose. D) This patient does not require any additional therapy. E) This patient is an ideal candidate for an oral bisphosphonate.

Answer A. This patient should receive yearly zoledronic acid. Explanation This patient with osteopenia has several additional risk factors for the progression to osteoporosis such as postmenopausal, levothyroxine use, and a recent fracture. Because her T-score is below -1.5, she should undergo additional osteoporosis therapy. While the standard of care is bisphosphonate therapy, her severe GERD precludes an oral bisphosphonate. Calcitonin is rarely used in osteoporosis, except in the setting of an acute fracture, because it appears to provide some analgesia. No data exists that decreasing levothyroxine improves bone mineral density. However, intravenous bisphosphonate therapy, zoledronic acid, is the appropriate choice for this patient. Intravenous ibandronate would have also been correct.

A 47-year-old laborer is complaining of numbness and tingling of his right hand, which wakes him up at night. He is on glyburide for his diabetes. He has a history of apparent gout and has taken colchicine intermittently. He injured his left knee in a motor vehicle accident several years ago and has mild pain. Examination reveals no wasting of the right thenar eminence. Sensory deficit is noted of the thumb and index finger. Tinel's sign is positive. Crepitus of the right knee is present, but there is no swelling. Nerve conduction velocities reveal slowing across the carpal tunnel of the median nerve. Which of the following would not be appropriate? A) Vitamin B6 B) Right wrist splint C) Right carpal tunnel corticosteroid injection D) Carpal tunnel release

Answer A. Vitamin B6 Explanation The inappropriate choice is vitamin B6. The patient seems to have clinical evidence suggestive of carpal tunnel entrapment. Release, splint, and corticosteroid injection are all appropriate. Vitamin B6 has been studied and has been shown not to be effective

A 54-year-old Caucasian male presents for evaluation of an acute painful right ankle. Symptoms began 24 hours prior to his visit. He has never experienced joint pain in the past. Other than being overweight and hypertensive, he is healthy. After unsuccessful trials of exercise and salt restriction, he was started on hydrochlorothiazide 6 months ago. There is no family history of arthritis. He recalls no injury. Examination reveals: Temp 99.9° F (37.7° C), BP 160/92 mmHg, Pulse: 88 and regular, and RR: 12 and unlabored. Skin exam reveals no rashes or lesions. Cardiovascular exam reveals a regular rate and rhythm. An S 4 murmur is present. Pulmonary and abdominal exams are normal. Joint exam is significant for erythema and warmth of the right ankle. A small effusion is present. There is significant pain on attempted range of motion. Joint aspiration reveals translucent fluid with the following characteristics: WBC 4,500 cells/mm 3 60% PMNs T. protein 4.0 g/dL LDH 125 U/L (Serum 80 U/L) Glucose 40 g/dL (Serum 95 g/dL) Gram stain: no organisms Polarizing microscope: negatively birefringent, needle-shaped crystals Which of the following is the most appropriate management? A) Exclude underlying etiologies by evaluating the following labs: TSH, ferritin, and calcium level. B) After treating his acute attack, consider switching his antihypertensive treatment from HCTZ to a different antihypertensive. C) Draw a serum uric acid level to confirm the diagnosis. D) Prescribe NSAIDs and allopurinol.

Answer B. After treating his acute attack, consider switching his antihypertensive treatment from HCTZ to a different antihypertensive. Explanation Hydrochlorothiazide is known to cause retention of uric acid by stimulating a net increase in reabsorption of urate at the proximal tubule of the kidney. Given that the first attack of gout has occurred within six months of institution of hydrochlorothiazide, discontinuation of this medication is appropriate if an alternative treatment for hypertension can be instituted. The diagnosis of gout can be confirmed only by demonstrating the presence of intracellular negatively birefringent uric acid crystals on joint fluid analysis. The serum uric acid does not confirm this diagnosis. With diagnosis, appropriate treatment does include use of NSAIDs. Allopurinol, however, should never be instituted or discontinued in the setting of an acute gouty attack. Hypothyroidism, hemochromatosis, and hyperparathyroidism are three causes of calcium pyrophosphate crystal disease (CPPD or "pseudogout"). Pseudogout is associated with positively birefringent, rhomboid-shaped crystals in synovial fluid. The negatively birefringent crystals in his fluid establish his diagnosis definitively as gout (monosodium urate crystal deposition disease). Checking for TSH, ferritin, and calcium is not indicated with this clinical picture.

A 19-year-old Caucasian male is seen for low back pain and stiffness. He has been very used to running and playing basketball and is now finding it increasingly difficult to compete in these activities. For the past 18 months, he has developed progressive low back pain, which he wakes up with in the morning. Somewhat surprisingly, he notices that this pain improves on his early morning run. His previous medical history has been benign, except for occasional episodes of diarrhea. He has discussed this with his primary care physician and was told that this possibly was irritable bowel syndrome, because it seemed to be made worse by stress related to exams—which he was taking during his freshman year. On one occasion, he had fairly bloody stools, which lasted about 36 hours, but he failed to report this to his primary care physician. Family history is benign—no family members with arthritis or back problems. On examination, he is well nourished and well hydrated. The general exam is benign. His joints appear quite normal. His neck and lumbosacral movement is full and normal, and he has a negative straight leg-raising sign. Schober test reveals 6 cm of movement. There is no evidence of psoriasis. Lab investigations revealed a sed rate of 63, hemoglobin of 10.9, WBC of 7.6, and creatinine of 0.8. U/A is negative. Which of the following is the least appropriate test to order in this patient? A) MRI of the LS-spine B) CT scan of the SI joints C) Referral to ophthalmology for slit-lamp examination D) Colonoscopy

Answer B. CT scan of the SI joints Explanation The patient has a seronegative spondyloarthropathy probably due to inflammatory bowel disease (enteropathic spondyloarthropathy). A CT scan would be inappropriate. This patient has classic symptoms of inflammatory back pain, suggesting a spondyloarthropahty. An MRI of the SI joints has become the study of choice in the last decade. While early changes may be missed on a simple film of the LS-spine and SI joints and a CT scan is highly sensitive for erosive changes in the SI joints, most do not recommend a CT scan because of the increased gonadal radiation associated with this modality. MRI of the lumbosacral spine also would pick up a bamboo spine correlating with his abnormal Schober test. Sub-clinical uveitis needs to be excluded in this patient. His GI symptoms certainly suggest that he has inflammatory bowel disease, which definitely could be related to his musculoskeletal symptoms. Thus, the need for colonoscopy.

You see a 70-year-old man for a routine exam. He is in excellent health and takes no medications. Family history is unremarkable. He drinks alcohol only occasionally and does not smoke. ROS is positive for nocturia (twice each night). Patient also has had occasional constipation and some hearing loss over the past year, but otherwise appears normal. Physical exam: Healthy-appearing man Vital signs normal No lymphadenopathy Thyroid non-palpable Cardiac, pulmonary, abdominal exams normal No focal neurologic abnormalities No rash Prostate enlarged, symmetric, no nodules, nontender Laboratory results: AST: 30 U/L ALT: 32 U/L Alk Phos: 298 (36-92 U/L) Calcium: 8.6 mEq/L Phos: 2.2 mEq/L Mg: 2.8 mEq/L T. bili: 0.6 mg/dL PT: 11.0 s PTT: 20.8 s T. protein: 6.4 g/dL LDH: 80 (60-100 U/L) Albumin: 3.9 g/dL Urinalysis: Negative PSA: 4.2 (mildly elevated) Which of the following findings is likely to be present with further testing? A) Elevated parathyroid hormone levels B) Cotton-wool appearance on skull films C) Elevated parathyroid hormone-related peptide levels D) Monoclonal spike on SPEP, increased uptake on bone scan E) Extracapsular adenocarcinoma of the prostate on biopsy

Answer B. Cotton-wool appearance on skull films Explanation This is a case of Paget disease of bone. Paget's is often found on routine lab testing (elevated alkaline phosphatase), because it is usually asymptomatic. Calcium levels are usually normal. Paget lesions are metabolically active and would light up on bone scan, which tests for osteoblastic (not osteoclastic) activity; however, skull or long-bone films are likely to show the classic feature of Paget's: "osteitis deformans." Hearing loss may occur from bony involvement of the skull. In patients with widely metastatic prostate cancer, bone scan would show increased osteoblastic activity, but a higher PSA might be expected, and SPEP would not show a monoclonal spike. Patients with hyperparathyroidism and malignancy-related PTHrP elevations should have elevated serum calcium levels. Patients with multiple myeloma may have hypercalcemia and an abnormal SPEP, but myeloma rarely lights up on bone scan (myeloma is primarily an osteoclastic bone disease). Hand involvement with osteoarthritis is common.

An 80-year-old woman presents to your clinic with chronic back pain. She says that she is frequently short of breath. Also, when she walks to the grocery store, she frequently feels a pain in her chest. She describes the pain as a mild pressure. When she rests, the pain goes away. On physical examination, she has dorsal kyphosis with a protuberant lower abdomen. You are concerned that she may have osteoporosis. Which of the following is not a clinical feature of osteoporosis? A) Protuberant lower abdomen B) Exertional chest pain relieved with rest C) Back pain D) Pulmonary dysfunction E) Dorsal kyphosis

Answer B. Exertional chest pain relieved with rest Explanation Obviously, this woman is having pain, which must make you think about angina or coronary disease syndromes. All of the other findings are seen with osteoporosis. The pulmonary dysfunction can occur as the total lung capacity is reduced by changes in the architecture of the thorax due to disease progression and worsening dorsal kyphosis and cervical lordosis.

A 42-year-old gentleman presents with diffuse joint pain and swelling, which has been ongoing for the past 18 months. He initially began with some knee pain and swelling and was unable to continue with his usual habit of cycling. However, the episodes of joint pain were seemingly unrelated to his cycling, and he continued to have episodes of swelling that occurred very spontaneously that would last for 10-12 days and then subside. These paroxysmal episodes then began to involve his wrists and then the 2nd and 3rd MCP joints on both sides. He was seen by his primary care physician, who found him to have a sed rate of 53 and a negative RF and ANA. X-rays of the relevant joints were thought to be negative in the early stages. He never had an episode of podagra. He has no history of renal calculi. The rest of the systemic inquiry was benign. He has no family history of joint problems. There are no problems with his eyes. He tried several NSAIDs; these were generally unsuccessful. He was even prescribed a prednisone dose-pack equivalent. On examination, he had a BP of 112/76. General exam was benign. Musculoskeletal exam revealed some tenderness over the 2nd and 3rd MCP joints on both sides with no effusions or synovitis. His right wrist had somewhat limited flexion and extension. The knees were without effusions. Repeat lab investigations again found him to be RF-negative, and he had a normal sed rate. X-ray of the hands revealed significant osteophytes over the 2nd and 3rd MCP joints bilaterally with right wrist triangular fibrocartilage calcification on the right. Given the fact that there were no active joints to tap, which of the following investigations would be appropriate at this stage? A) CT scan of the SI joints B) Ferritin and calcium levels C) Anti-Jo-1 antibody D) Bone scan

Answer B. Ferritin and calcium levels Explanation The history of intermittent inflammatory arthritis and evidence of chondrocalcinosis suggest a diagnosis of calcium pyrophosphate deposition disease (CPPD). In the presence of hooked or bulky osteophytes in the 2nd and 3rd MCP joints, hemochromatosis should be considered. During an acute flare, synovial fluid may reveal CPPD crystals, which are positively birefringent. Hemochromatosis and hyperparathyroidism are 2 diseases strongly associated with CPPD. A CT scan of the SI joint would not be helpful because suspicion of a seronegative spondyloarthritis is low. A bone scan may show inflamed joints but would not be specific to help aid in diagnosis. Anti-Jo-1 occurs in the setting of myositis and would not be helpful in this case.

A 55-year-old man presents for routine physical examination. He gives a 10-year history of intermittent episodes of severe pain and swelling of the joints, occurring about every 3 to 5 months and lasting for about 1 week. He says these episodes "are just like my pappy has." Between the attacks, he has virtually no joint pain. His last attack was about 2 months ago, and he is without symptoms when he sees you today. PAST MEDICAL HISTORY: Essentially negative FAMILY HISTORY: His father is 70 years old with similar joint complaints SOCIAL HISTORY: Lives alone with his 2 cats Doesn't smoke Drinks a 6-pack of beer daily PHYSICAL EXAMINATION: Extremities: Hallux valgus (bunion) deformity of both 1st metatarsophalangeal joints Firm, enlargement of the right 2nd and 4th proximal interphalangeal joints and the left 1st and 5th proximal interphalangeal joints Several hard nodules are palpated in the left olecranon bursa, which is swollen but not tender, not warm, and not erythematous An x-ray of the right foot shows soft tissue density around the 1st metatarsophalangeal joint and an oval bone erosion with an overhanging edge in the 1st metatarsal bone at the metatarsophalangeal joint. Which of the following is the most likely cause of his complaints? A) Pseudogout B) Gout C) Osteoarthritis D) Rheumatoid arthritis E) Tendinosis universalis

Answer B. Gout Explanation His x-ray findings are consistent with long-standing gout. Remember, the 1st metatarsophalangeal joint is the most common joint to be involved. Also, he has chronic tophaceous gout with the findings of the tophaceous deposits of monosodium urate (negatively birefringent crystals!) at his olecranon bursae. The other sites for these are the Achilles tendons and the external ear. Tophi are painless, but their presence in and around joints can limit joint mobility.

An 80-year-old woman with rheumatoid arthritis for 30 years presents for follow-up. She has had an extensive history and has developed many complications from her disease. Which of the following is not a characteristic deformity associated with rheumatoid arthritis? A) Ulnar deviation B) Heberden node C) Boutonniere deformity D) Hammer toe E) Swan-neck deformity

Answer B. Heberden node Explanation Heberden and Bouchard nodes are not seen in rheumatoid arthritis but are seen in osteoarthritis. Heberden nodes are enlargement of the DIP joint, and Bouchard nodes are bony enlargement of the PIP joints. All of the other findings listed are consistent with long-standing rheumatoid arthritis.

A patient is treated with prednisone 20 mg qd for what was thought to be CNS cerebritis as part of her lupus. Her prednisone dose was slowly tapered over 2 months. Approximately 5 months after this, she returns with a fairly sharp headache, mild joint pain, and a fever. She is complaining of left-sided weakness with her leg feeling weaker than the arm. She had gone to sleep the previous night feeling quite well except for a mild headache, but woke up at 4:00 a.m. with a fairly sharp headache and a sense of weakness on the left as described above. When seen in the emergency department, it was fairly apparent that she had a left hemiparesis. She had no other new findings of note. A repeat CT scan of the brain is entirely normal; this was done without any contrast. At this time, her WBC is 2.7, hemoglobin is 11, and platelet count is 62,000. The patient also has a positive lupus anticoagulant (LA) and elevated anti-cardiolipin IgG antibodies. Which of the following is the most appropriate management at this stage? A) Methylprednisolone sodium succinate (Solu-Medrol®) 1 g IV for 3 consecutive days. B) Initiate IV heparin and schedule MRI of the brain. C) IV cyclophosphamide q month. D) Low-dose aspirin.

Answer B. Initiate IV heparin and schedule MRI of the brain. Explanation She needs an MRI of the brain with contrast. It is likely that an infarct will be demonstrated. She has previously been known to have a positive lupus anticoagulant. Look for anticardiolipin antibodies. IV methylprednisolone sodium succinate would be given for severe CNS cerebritis, particularly if this was thought to be potentially life-threatening. At this time, it is very contentious for how long she should be anticoagulated, but her INR should be maintained between 2 and 3; low-dose aspirin therapy is ineffective. IV cyclophosphamide has no role in CNS infarct.

A 20-year-old male presents to clinic with knee pain. The patient is a college student and was playing basketball 3 days ago. He landed awkwardly, fell, and had immediate pain in his right knee. He was able to walk off the court on his own. The following day, he awoke to find his knee swollen and painful. He was able to walk, but was unable to practice and so is presenting to clinic. His past medical history is remarkable for exercise-induced asthma. He does not smoke, drinks on weekends, and is sexually active with one partner, with a lifetime history of 2 partners. Physical exam is remarkable for a swollen right knee with a ballotable patella. The knee is not red or warm. He has no joint line tenderness. Lachman and anterior drawer testing are normal. The next step in management of the patient is: A) Knee immobilization, referral to orthopedic surgery B) Knee immobilization, rest, ice, elevation, and ibuprofen 800 mg three times a day C) Urine PCR for gonorrhea D) Knee immobilization, MRI of the knee

Answer B. Knee immobilization, rest, ice, elevation, and ibuprofen 800 mg three times a day Explanation The patient has a knee injury consistent with a meniscal tear, with the ability to bear weight after the injury, delayed onset of swelling, and an otherwise normal knee exam. Conservative treatment with a progressive return to activity is most appropriate. An MRI of the knee and orthopedics referral would be appropriate if the patient had locking, clicking, or further disability following the injury. While disseminated gonococcal infection is on the differential diagnosis for a young adult with monoarticular arthritis, the patient had a clear antecedent injury and has a low-risk sexual history.

A 50-year-old man swims daily in a lake. Approximately 2 years ago he injured his left foot while in the lake after stepping on some rocks. Pain and swelling developed at the site of injury, which was just proximal to the 3rd digit on his left foot. Traumatic synovitis was diagnosed, and he was started on oral ibuprofen and local corticosteroid injections. During the next 2 years, the pain and swelling have persisted, and now have spread to the plantar surface of his foot and to his ankle. He has not had any fever or chills. Three synovectomies have been performed, and each showed non-caseating granulomas. Special stains revealed no mycobacteria or fungi. Cultures have not grown anything at 2 months since the last synovectomy. He comes to you today because it is worsening again. Which of the following organisms is most likely causing his infection? A) Mycobacterium tuberculosis B) Mycobacterium marinum C) Sporothrix schenckii D) Nocardia brasiliensis E) Blastomyces dermatitidis

Answer B. Mycobacterium marinum Explanation Note that he has a chronic process x 2 years. He sustained an injury in water. Put the two together and you come up with Mycobacterium marinum! Note it is common for you not to be able to culture this organism. Again, the water exposure is the kicker here. Think of the other organisms if the environmental exposure is right: Nocardia—soil; Blastomyces—soil or lumber; Sporothrix—gardens or plants.

A 60-year-old man presents with complaints of left knee pain. He says that he has had chronic bilateral hip, knee, and shoulder pain for many months. These joints are usually "stiff" in the morning but improve after 20 minutes. His left knee became swollen 3 days ago, and this is new. He cannot bear weight on this leg without severe pain in his knee. His physical examination shows a swollen, red, warm left knee. He has a palpable effusion of the affected knee. The neurological, vascular, and sensory functions of the knee area are intact. You aspirate the knee and it shows 1,800 WBC/µL with 95% lymphocytes. No crystals or other abnormalities are noted. What is the probable etiology of his left knee findings? A) Rheumatoid arthritis B) Osteoarthritis C) Septic arthritis D) Gout E) Pseudogout

Answer B. Osteoarthritis Explanation Osteoarthritis, also known as Degenerative joint disease (DJD), is the only diagnosis listed that would present with a non-inflammatory synovial fluid of 1,800-2,000 WBC, mostly consisting of lymphocytes. Gout, pseudogout, and RA will have an inflammatory synovial fluid with WBCs in the range of 2,000 - 50,000 PMNs. Septic arthritis will have > 50,000 WBC, almost all PMNs. The morning stiffness lasted less than 30 minutes, making RA also much less likely. The lack of crystals was helpful to rule out gout or pseudogout.

Radiographs of a patient's hands show narrowed joint spaces of the DIP and PIP joints without erosions or chondrocalcinosis. Which of the following is the most likely diagnosis? A) Psoriatic arthritis B) Osteoarthritis C) Chronic polyarticular gout D) Hemochromatosis E) Rheumatoid arthritis

Answer B. Osteoarthritis Explanation This question is based on straightforward recall, and on occasion, the Boards will give you a few of these. The arthritis associated with PIP and DIP joints without chondrocalcinosis or significant erosions is most likely to be OA. RA and psoriasis are erosive diseases. Hemochromatosis is associated with CPPD. Gout does not typically affect PIP and DIP joints.

A 50-year-old diabetic female presents with left shoulder pain and stiffness, which has been progressive over the past several weeks. The pain wakes her up at night. She had a pacemaker placed 3 months ago, having suffered an anterior myocardial infarction 6 months ago. Examination: BP = 137/87 Pulse = 76 Musculoskeletal exam reveals limited ROM of the left shoulder with abduction to 90 degrees, flexion to 100 degrees, and minimal extension. The right shoulder has full ROM. Mild lower-extremity sensory deficit. Mild sclerodactyl of the hands are noted. There is no swelling of the joints. Which of the following would be an appropriate treatment? A) Left subacromial corticosteroid injection B) Physical therapy C) Non-steroidals D) Prednisone 15 mg

Answer B. Physical therapy Explanation The patient has several risk factors for frozen shoulder syndrome, and the appropriate treatment is aggressive physical therapy. Perhaps non-steroidals could be used, but physical therapy is the first option. A steroid injection would not help this adhesive capsulitis. Prednisone is not indicated and would worsen this patient's diabetes.

A 15-year-old male runner comes to your clinic with the complaint of left heel pain off and on for a month. He is on the track team and runs cross-country. There is no known injury to his foot. The heel pain occurs mainly when he first gets out of bed in the morning and when he runs. It is located on the medial aspect of the heel. On physical exam, there is no redness, swelling, or significant tenderness. Gait is normal. There is no tenderness at the insertion of the Achilles tendon into the calcaneus. What is the most likely diagnosis? A) Achilles tendonitis B) Plantar fasciitis C) Calcaneal apophysitis D) Fracture E) Sprain

Answer B. Plantar fasciitis Explanation This is inflammation of the supporting structure (plantar fascia) of the longitudinal arch. It manifests as medial heel pain. The pain is worse when running and when taking the first few steps in the morning after getting out of bed. Treatment involves stretching of the gastrocsoleus, nonsteroidal antiinflammatory agents, and arch supports. Sometimes regional steroid injections are helpful. Runners may have to cut back on their mileage.

Consider a 72-year-old woman with known mild osteoarthritis and HTN, who consistently takes a thiazide diuretic and occasional celecoxib 200 mg qd for hand and knee pain. She has otherwise been in excellent health. She is admitted for a right lower lobe pneumonia and prescribed IV antibiotics. On the third day of her admission, she presents with an acutely swollen, tender, and painful right knee. This is the first time that any joint has actually swollen to such an extent. There is no history of podagra. Her uric acid level has always been normal. There is no history of renal calculi. On exam she is alert, well-oriented and well-hydrated. BP 162/76, T 99.7. MSK exam reveals asymmetric Heberden's nodes and a large right knee effusion. The right knee effusion is tapped. Which of the following findings are most likely in this clinical setting? A) Synovial fluid that is a transudate with a WBC of 2,200 B) Positively birefringent crystals C) Negatively birefringent crystals D) Calcium oxalate crystals

Answer B. Positively birefringent crystals Explanation The patient most probably has pseudogout (with crystals of calcium pyrophosphate to be found). This is the classic situation in which there is a precipitating event such as an infection and/or surgery or trauma. The crystals would be mildly positively birefringent, which means the colors would be very light blue or yellow depending on the direction of the polarizing light. Gout (negatively birefringent crystals) is possible since she is on a thiazide diuretic, but she has had normal uric acid levels and has never had podagra. Nevertheless, this would be less likely than pseudogout.

A 45-year-old woman presents with arthritis, malar rash, Raynaud phenomenon, leukopenia, and photosensitivity. She is worried that she may have lupus. According to current guidelines, which of the following is not a clinical manifestation used to diagnose systemic lupus erythematosus? A) Photosensitivity B) Raynaud phenomenon C) Leukopenia D) Malar rash E) Arthritis

Answer B. Raynaud phenomenon Explanation Raynaud phenomenon is not used as a criterion to classify systemic lupus erythematosus. Be sure you know the ACR classification criteria for SLE. On the exam, they will give you a patient with nonspecific findings, and you will have to know to find 4 or more of the 11 that are SLE-specific. The 11 are: photosensitivity, blood changes (anemia, leukopenia, etc.), renal disease, +ANA, +anti-dsDNA or anti-Sm antibody or antiphospholipid antibody, arthritis, malar rash, discoid rash, neuropsychiatric features (psychosis, seizures), oral lesions, and serositis.

A 20-year-old student is referred to your office from campus health care. There is a concern that she has a possible connective tissue disease process. She lives at home in Florida during the summer months but attends school in the Northeast during the winter months. Over the last month or two of severe winter weather, she has noticed that her fingers seem to be quite painful and go through some color changes that are brand new to her. Most specifically, they are initially white and then seemingly go a little blue and then remain red for prolonged periods of time. She struggles to complete her projects because she finds it very difficult to type. She thinks her finger joints are a little painful. She readily admits to some alcohol intake and has recently begun to smoke 5-10 cigarettes per day, particularly more so over the weekends. She denies any rashes, swallowing problems, shortness of breath, or cough. She has no problem swallowing food. As part of her workup on campus, she was found to have a strongly positive anti-nuclear antibody. She had been reading about lupus and was very concerned about this condition, particularly from the information that she had gleaned from the Internet. On examination, she is alert and oriented. BP is 112/57, pulse is 76. All pulses are present and equal. She has no bruits. There is a positive Allen's test. Cardiorespiratory exam is benign. Musculoskeletal exam is benign. Cutaneous exam reveals no rashes, vasculitis, or pulp atrophy. There is a negative vital nailfold capillaroscopy. Lab findings include a positive ANA in a titer of 1:320 (normal is less than 1:40). Hemoglobin is 11.9, and WBC is 4.9. U/A is negative. Which of the following would be appropriate for further workup? A) ECHO to estimate pulmonary pressure B) Repeat ANA, anti-ds DNA titers, anti-centromere, and anti-SCL70 C) Bronchoscopy D) Upper extremity arteriography E) Esophageal manometry

Answer B. Repeat ANA, anti-ds DNA titers, anti-centromere, and anti-SCL70 Explanation The patient most likely has primary Raynaud disease. Approximately 20% of younger women with primary Raynaud's have a positive ANA, and this is not always an appropriate screening test. However, given the fact that her ANA is known to be positive, it would be appropriate and reassuring to exclude a secondary process. Her symptoms do not particularly suggest a connective tissue disease process, but a repeat ANA would be appropriate because of variations from lab to lab. The specific serologies for lupus, including anti-ds DNA and SM, would be reasonable. So, too, would be an anti-centromere to rule out CREST and an anti-SCL70 to exclude diffuse systemic sclerosis. If these last 2 tests were found to be positive, then a further screening to include manometry, ECHO, etc., would be appropriate. The negative vital capillaroscopy and normal Allen's test (positive blood flow) is indeed reassuring.

A 40-year-old woman presents for a follow-up appointment. Last month you evaluated her for complaints of persistent joint pains. At that time she had symmetrical swelling of her wrists, MCPs, and PIPs. You placed her on a nonsteroidal antiinflammatory agent that provided some relief, but she still has similar complaints. Her physical examination today is no different from last month's. An x-ray of the hands shows erosive synovitis in the MCP and PIP joints of both hands. Which of the following is not consistent with a diagnosis of rheumatoid arthritis? A) Mononeuritis multiplex B) Sacroiliitis C) Erosive synovitis changes of her hands on x-ray D) Swelling of the wrist, MCPs, or PIPs for 6 weeks E) Morning stiffness for longer than 45 minutes for 6 weeks

Answer B. Sacroiliitis Explanation All of the findings except sacroiliitis are associated with the diagnosis of rheumatoid arthritis (RA). The finding of mononeuritis multiplex with wrist or footdrop is also associated with RA. The symmetrical swellings described, erosive x-ray findings, prolonged morning stiffness and joint pains > 6 weeks duration are all characteristic clinical features of RA.

A 78-year-old woman comes to your office because of severe headache. She has also had muscle aches in her thighs, loss of appetite, malaise, and experiences jaw pain when eating or talking for a long period of time. On exam, she has no nuchal signs but does have tenderness over the temporal region. There are no focal findings on her neurological examination. Which test will help to make a definitive diagnosis? A) MRI of the brain B) Temporal artery biopsy C) Lumbar puncture D) Erythrocyte sedimentation rate (ESR) E) CT of the brain

Answer B. Temporal artery biopsy Explanation The most likely diagnosis is temporal arteritis, a granulomatous inflammation that affects the medium and large vessels in persons over 55 years of age. Associated physical symptoms include those mentioned in the history; jaw claudication is present in > 50% of affected individuals. Although an MRI may be ordered to narrow the differential, and the ESR is typically >100, the biopsy will show multinucleated giant cells, the pathological hallmark of this illness. CT scan and lumbar puncture will not provide any information.

A 69-year-old woman presents with symptoms of severe muscle pain and joint pain. This has been present for the past 3 weeks. She has had no fevers, chills, or trauma. She has a past history of HTN, hypothyroidism, CAD, osteoporosis, GERD, and depression. Meds: omeprazole, metoprolol, alendronate, citalopram, levothyroxine. What is the most likely cause of her pain? A) Citalopram B) Metoprolol C) Alendronate D) Omeprazole E) Hypothyroidism

Answer C. Alendronate Explanation This patient has generalized muscle and joint pain. It fits well the syndrome of generalized pain that can occur with potent bisphosphonates. It is much more common with weekly and monthly dosing.

A 19-year-old male presents with low back pain and stiffness. These symptoms have been of gradual onset over the past 6 months. In general, he finds that a shower in the morning tends to make him feel less stiff, and once he starts walking around, the stiffness seems to disappear. There is no family history of consequence. His other peripheral joints have not been a problem, but more recently he has begun to notice some pain in the left groin area. There is no history of urethritis or of conjunctivitis. He has had no rashes. Otherwise, he has been in excellent health. The patient is well-appearing. His peripheral joints are benign. Schober test reveals 3 cm of movement. Cutaneous exam reveals no vasculitis, but some pitting is noticed on several nails and onycholysis of several others. Investigations reveal a sedimentation rate of 97, normal CBC and CMP, negative ANA and RF. X-ray of the lumbar spine reveals bilateral asymmetric SI joint sclerosis and narrowing. X-ray of the hips is normal. Which is the drug of choice? A) Hydroxychloroquine sulfate B) Prednisone 60 mg qd C) An anti-TNF agent D) Azulfidine E) Methotrexate

Answer C. An anti-TNF agent Explanation In central axial spinal involvement, the only drugs that have been demonstrated in randomized clinical trials to be beneficial are the anti-TNF agents. These represent the drug of choice. In general, the seronegative spondyloarthropathies do not tend to respond to systemic corticosteroids. Systemic corticosteroids also increase the risk of osteoporosis and fracture. DMARDS are not recommedned for axial disease. He probably has psoriatic arthritis, given the specific nail changes.

A 30-year-old man has had recurrent episodes of asymmetric inflammatory oligoarticular arthritis involving his knees, ankles, and elbows. The arthritis usually lasts anywhere from 2 to 4 weeks. Since age 20 or so, he has had recurrent painful "sores" in his mouth. Today he presents to you with fever, arthritis, and mild abdominal pain. Additionally, he reports a "severe" headache. Physical examination shows significant arthritis of his bilateral knees with warmth to touch and gross fluid accumulation. Additionally, you find a superficial thrombophlebitis in his left leg. ANA is negative. Which of the following is the most likely diagnosis? A) Ulcerative colitis examination shows significant arthritis of his bilateral knees. B) Crohn disease examination shows significant arthritis of his bilateral knees. C) Behçet syndrome examination shows significant arthritis of his bilateral knees. D) Whipple disease examination shows significant arthritis of his bilateral knees. E) SLE examination shows significant arthritis of his bilateral knees.

Answer C. Behçet syndrome examination shows significant arthritis of his bilateral knees. Explanation Behçet syndrome is a recurrent disease of unknown etiology and is characterized by painful oral and genital ulcers, eye inflammation, arthritis; CNS symptoms (aseptic meningitis) with fever, abdominal complaints and thombophlebitis also can be seen in this disease. These symptoms can mimic inflammatory bowel disease (IBD). While peripheral neuropathies are common with IBD, CNS involvement is nọt. Whipple disease is associated with arthritis, abdominal pain, and CNS disease, but typically seen in older males. A negative ANA virtually excludes the diagnosis of SLE.

A 58-year-old Caucasian female presents with aching of her shoulders and the sides of her hips. This has been present for the past 3-4 weeks. She has not wanted to go to the gym where she participates in exercise and water aerobics 3-4 times weekly. Her fingers, toes, knees, and elbows have not been painful. She has a history of HTN, which has been treated with a beta-blocker. There is no family history of joint problems. On examination, she is well nourished and well hydrated with normal BP and pulse of 76. Exam is completely without any specific findings. Her musculoskeletal exam is entirely negative without any loss of joint range or mobility. She has no synovitis in any joints and no tender trigger points. Lab investigations reveal a mild anemia with hemoglobin of 10.2, MCV of 88, and ESR of 99. She is placed on prednisone 15 mg qd. Which of the following is appropriate adjunctive therapy? A) Calcitonin B) Estrogen therapy C) Bisphosphonate D) A selective estrogen receptor modulator (SERM)

Answer C. Bisphosphonate Explanation She has polymyalgia rheumatica (PMR). Any patient who is going to be on prednisone at a dose of > 5 mg qd for longer than 3 months should be placed on a bisphosphonate as prophylaxis against glucocorticoid-induced osteoporosis (GIOP) with potential for fractures.These are current American College of Rheumatology recommendations for GIOP. For most PMR patients, the average patient requires glucocorticoid therapy for about 1 year. SERMs and calcitonin have not clinically been demonstrated to prevent glucocorticoid-induced osteoporosis. Estrogen would help but is no longer recommended because of the increased risk of breast cancer and stroke.

A 66-year-old female with HTN presents complaining of pain in her neck, shoulders, hips, and thighs x 2 months. She takes longer to dress in the mornings because of pain, and she is not sleeping well. She denies weakness and swelling of any joints or muscle groups. Exam shows a 12-lb weight loss since her last visit 4 months ago, but she is otherwise normal. WBC: 9,500 (normal differential) Hgb: 10.5 Platelets: 450,000 Na: 145 K: 4.2 HCO3: 24 BUN: 12 Creat : 0.6 Glu: 92 CPK: 70 (50-170) Which of the following is the most appropriate next step in management? A) Peroneal nerve biopsy B) Acetylcholine receptor antibodies C) Pregabalin daily D) Low-dose prednisone daily E) Anti-Jo antibodies

Answer D. Low-dose prednisone daily Explanation On the Board exam, the questions are not identified by section. Differentiating rheum conditions from neurologic conditions can be a real challenge. Regardless of what section you're studying, get in the habit of constructing a problem list and looking for patterns/illness scripts. The problem list for this patient: Pain (not weakness!) in the shoulders, hips, and thighs in an elderly woman causing sleep disturbance 10-pound weight loss Anemia and thrombocytosis Normal examination Normal CPK For what illness does this script fit? Polymyalgia rheumatica. The options ask you to choose from myasthenia gravis (acetylcholine receptor antibodies), a myositis (anti-Jo antibodies associated with polymyositis; colonoscopy might be done to diagnose an occult malignancy associated with dermatomyositis; and peroneal nerve biopsies help diagnose confusing cases of myositis), and empiric treatment for fibromyalgia or polymyalgia rheumatica (PMR). Be prepared to see PMR presented on the exam and juxtaposed among these diseases in the differential diagnosis. This case is not myasthenia because the presentation is pain, not weakness, and the patient profile is elderly female. Any shoulder pain in an elderly female should immediately suggest PMR. Myasthenia classically worsens with use and improves with rest. Anti-acetylcholine receptor antibodies are present in most cases of myasthenia and can be used to diagnose the condition (replacing the Tensilon® test). Myositis should be considered, but the normal CPK excludes it on the Boards. Fibromyalgia is a disease associated with abnormal sleep patterns and diffuse body aches; but on the Board exam, fibromyalgia will probably never be the right answer for any complaints in an elderly woman. This woman also has danger signs: anemia, thrombocytosis, and weight loss. These signs and symptoms suggest a true underlying serious disease process, such as a vasculitis, and are not associated with fibromyalgia. In practice, you might perform other tests (such as a TB skin test) before instituting systemic steroids. You might even refer this patient to a rheumatologist because long-term use of systemic steroids in an elderly woman is a rather serious treatment if you aren't completely comfortable with the diagnosis. The Board exam, however, may ask you to commit to empiric treatment before you are necessarily comfortable with a diagnosis. If empiric treatment is the best answer out of the choices listed, then don't fret about it. It's not a real patient; it's just a Board question. Choose the best possible answer and move on.

A 42-year-old machinist presents to the clinic with a history of 6 weeks of nagging mid-abdominal pain. He has not been previously ill. You discover that his peripheral eosinophil count is 4% and that his urine sediment is "telescopic" with red cells, white cells, and granular casts. Visceral angiography suggests the correct diagnosis, which is confirmed by biopsy that demonstrates involvement of medium-sized vessels showing the characteristic ring of acellular azurophilic fibrinoid necrosis. All tests for anti-neutrophil cytoplasmic antibody (ANCA) markers are negative, as is a hepatitis screen. Which of the following is the most likely diagnosis? A) Churg-Strauss syndrome B) Kawasaki disease C) Classic polyarteritis nodosa (PAN) D) Granulomatosis with polyangiitis (Wegener's) E) Microscopic polyarteritis (MPA)

Answer C. Classic polyarteritis nodosa (PAN) Explanation Visceral angiography may be necessary to establish this diagnosis. Biopsy of skin, kidney, or other tissues should demonstrate the typical fibrinoid necrosis. Although often suggested otherwise in various sources, the association of the ANCA antibodies is not with this entity but with the following: p-ANCA 1) Microscopic polyarteritis (MPA): involvement of venules, capillaries, arterioles 2) Churg-Strauss: eosinophilia c-ANCA 1) Granulomatosis with polyangiitis (Wegener's): upper respiratory involvement (sinuses) Classic PAN affecting medium-sized arteries is usually not associated with ANCA markers at all. If an antibody association is present, there may be evidence for hepatitis response in some patients.

A 74-year-old female presents to you with a 2-month history of bone pain, weight loss, and night sweats. She is very concerned about a red swelling over her left index finger. She has a history of NIDDM, coronary artery disease, and renal disease. PHYSICAL EXAMINATION: Well-nourished and alert woman who appears to be her stated age. Neck supple. No lymphadenopathy. Mild splenomegaly is noted. Asymmetric Heberden and Bouchard nodes noted with erythema and profound tenderness over left 2nd index DIP. LABORATORY: ESR = 118, Hgb = 10.0, Creatinine = 2.4 Serum protein electrophoresis reveals a monoclonal spike. Urine is positive for Bence-Jones protein. X-rays of her left hand show several osteophytes at the DIPs and PIPs but large marginal erosions at the 2nd DIP joint. What is the cause of her acute left finger symptoms? A) Osteoarthritis B) Amyloid C) Gout D) Ochronosis E) Pseudogout

Answer C. Gout Explanation She has multiple myeloma and osteoarthritis. She has a fairly typical presentation of acute gout in the setting of a non-benign disease. The unusual feature is that her gouty attack is taking place in her Heberden node. This is identified by the characteristic feature of "marginal erosions" typical of gouty arthritis.

A 50-year-old man presents with acute swelling of his first metatarsophalangeal joint. He admits to excessive alcohol intake and is on hydrochlorothiazide for hypertension. He is obese. You tap the joint and discover urate crystals in the joint fluid. Uric acid levels are normal. What is the most likely diagnosis? A) Without knowing the birefringence, unable to assess if this is pseudogout or gout. B) Because of his age, alcohol intake, and normal uric acid levels, pseudogout is more likely. C) Gout has definitively been diagnosed. D) Because of his age, alcohol intake, and normal uric acid levels, hemochromatosis is more likely. E) Septic arthritis is most likely.

Answer C. Gout has definitively been diagnosed. Explanation Urate crystals in the joint fluid are diagnostic for gout. Period. His age, obesity, podagra, and alcohol intake, as well as his diuretic use, improve the pre-test probability that you will find crystals; but in any case, finding urate crystals is diagnostic. You do not need the birefringent properties to make the diagnosis here. They've told you that urate crystals are present.

A 63-year-old female returns to your clinic for her annual physical examination. She is doing very well with her only ongoing complaint being chronic knee pain. Her condition has been diagnosed as osteoarthritis, and her complaints have been well controlled with acetaminophen. Which of the following is the most likely side effect with this therapy? A) Constipation B) Gastroesophagitis C) Hepatitis D) Glomerulonephritis E) Abnormal bleeding

Answer C. Hepatitis Explanation Mild and moderate osteoarthritis can be very effectively treated with acetaminophen with a minimization of side effects commonly linked with nonsteroidal antiinflammatory agents and narcotics. Treatment with nonsteroidal antiinflammatories increases the risk of gastroesophagitis, renal impairment, and mildly increases the risk for abnormal bleeding. Narcotics predispose to the development of constipation. Treatment with acetaminophen does raise the risk of hepatic injury with either chronic or high-dose treatment courses.

You suspect a woman with recurrent spontaneous abortions has the antiphospholipid syndrome. What do you expect her coagulation studies to show? A) She will have Factor VII deficiency. B) Lupus anticoagulant does not affect coagulation. C) Her PTT will not correct when her plasma is mixed with normal, but will correct with the addition of excess phospholipid. D) Her PTT will be normal. E) Her PTT will correct when her plasma is mixed with normal.

Answer C. Her PTT will not correct when her plasma is mixed with normal, but will correct with the addition of excess phospholipid. Explanation Prolongation of the partial thromboplastin time (PTT) that is not corrected by adding 1:1 mix of normal plasma, but does correct after the addition of excess phospholipid. suggests the presence of a lupus anticoagulant. The coagulation studies (PTT) will fail to correct because the coagulation factor inhibitors will inhibit the coagulation factors in the normal plasma also.

A 53-year-old African-American gentleman presents with profound weakness and minimal muscle pain. These symptoms began 2-3 weeks ago. Initially, he could no longer weight-train in the way he was accustomed. He is not short of breath or coughing. He denies any rashes or joint pain. His energy levels have been dramatically reduced. He is having no problems with swallowing. On examination, he is quite weak proximally. He is barely able to rise from a seated position. Proximal muscles in the upper extremities are similarly weakened, but distal muscle strength is normal, and there are no sensory deficits detected. Vital signs are normal. The joints have a good range of motion and are without active synovitis. Cutaneous exam is positive both for a peri-orbital heliotrope rash and Gottron plaques on the extremities. The remainder of his examination is normal. Lab findings reveal hemoglobin of 11.3, WBC of 12.1, and normal platelet count. Creatinine is normal. Creatinine kinase is 19,512. A muscle biopsy reveals fairly non-specific lymphocytic and neutrophil infiltrates of some of the muscle fibers, with occasional areas of necrosis. Which of the following therapies would not be appropriate now? A) 60 mg of prednisone B) Alendronate 70 mg q week C) IV immunoglobulin D) 1,000 mg of elemental calcium per day in divided doses

Answer C. IV immunoglobulin Explanation The "incorrect" therapy would be IV immunoglobulin. IVIG is indicated for refractory cases of dermatomyositis, not as initial therapy. This patient obviously has dermatomyositis and will require long-term corticosteroid therapy. According to the latest recommendations of the American College of Rheumatology, any patient requiring prednisone at a dose of 5 mg qd or greater for a period of 3 months or longer will require a bisphosphonate. Bone loss is dramatic in the first few weeks after initiating glucocorticoids, particularly at higher doses. However, dramatic bone loss occurs even at a relatively low dose of 7.5 mg qd. Calcium is also used as adjunctive therapy with bisphosphonates.

A 45-year-old woman presents with intermittent wrist and hand pain for 3 weeks. She has a history of DM and HTN, and her medications currently include glyburide and a diuretic. Exam: afebrile, R-wrist effusion without heat or erythema. She appears tan, and it is December in Minnesota. Radiograph of her hand shows the following: Which of the following is the most appropriate next step in patient care? A) Discontinue HCTZ and start enalapril. B) NSAID and wrist splint. C) Iron, TIBC, ferritin. D) ANA. E) Serum uric acid level.

Answer C. Iron, TIBC, ferritin. Explanation Exam in this case describes unilateral synovitis. The patient also has one glaring chronic disease: diabetes. On the Boards, when you see diabetes and joint pain in the same question, immediately consider hemochromatosis and CPPD. Of course, in real life, diabetics have lots of reasons for joint pain, but we're preparing you for Boards here. In this question, crystals are not described or shown, but a radiograph demonstrates chondrocalcinosis in several sites (wrist, between ring finger and pinky, and around the PIP of the long finger). Chondrocalcinosis is a finding commonly associated with CPPD. Hence, in a patient with diabetes and CPPD, it is appropriate to evaluate for underlying hemochromatosis (iron, TIBC, and ferritin). Also look out for words like "tan" and "dark-skinned" associated with any patient who has diabetes. Any question describing a "bronze diabetic" is screaming "hemochromatosis!"

A 33-year-old Caucasian female presents with diffuse arthralgias in an outpatient clinic. She reports joint swelling, stiffness, and fatigue. Symptoms have been ongoing since childhood. She has seen specialists in the past without a diagnosis. She self-medicates with ibuprofen daily with partial relief. The pain does not wake her up from sleep. On physical exam, she has velvety skin. She can hyperextend her fingers, elbows, and knees. Despite having back pain, she has full ROM of her lumbar spine and can touch the floor with the palms of her hands and do the splits easily. She had 2 tender points on exam. There is no joint swelling. What is her likely diagnosis? A) Seronegative spondyloarthritis B) Rheumatoid arthritis C) Joint hypermobility syndrome D) Osteoarthritis E) Fibromyalgia

Answer C. Joint hypermobility syndrome Explanation This patient fulfills Beighton and Brighton criteria for joint hypermobility syndrome—the Beighton score is based on: Being able to bend the thumb to the forearm (1 point each per thumb) Hyperextension of the fingers (1 point each for right and left hand) Hyperextension of the elbows (1 point each per elbow) Hyperextension of the knees (1 point each per knee) Being able to touch the palms to the floor while standing with knees extended (1 point) A score of 4/9 or higher suggests hypermobility. This patient's history and exam do not suggest an inflammatory process, thus excluding rheumatoid arthritis and seronegative spondyloarthritis. She also does not fulfill criteria for fibromyalgia. While she may develop secondary osteoarthritis due to joint hypermobility, this is not the best answer for the case scenario.

A 20-year-old man presents to clinic with back pain and morning stiffness. He has had these symptoms for about 8 months. In the beginning, he tried over-the-counter medications, but they did not help. His pain was initially in the lower back area and seemed to improve with exercise or movement. He has no radiation of the pain. He has had "itchy eyes" but seems to think these are normal with his allergies this time of year. His physical examination is significant for slightly injected conjunctiva. He has a II/VI early diastolic murmur heard best at the 2nd left intercostal space. His musculoskeletal examination shows no active synovitis. He is tender over both of his sacroiliac joints but has normal lumbo-sacral movement. He has a negative straight leg-raising sign. He does not have psoriasis. Which of the following should you do next to confirm the diagnosis? A) HLA-B27 serology B) Echocardiogram C) MRI of the sacroiliac joints D) Chest x-ray E) Ophthalmology consult

Answer C. MRI of the sacroiliac joints Explanation He has signs and symptoms of early ankylosing spondylitis. The other items listed would be done but would not be diagnostic for his disease process. The HLA-B27 is highly sensitive, but is not 100% specific for sacroiliitis as 8% of normal healthy Caucasians are HLA-B27 positive. The x-rays would most likely be normal in early ankylosing spondylitis. MRI is more sensitive than x-rays and shows "marrow edema" in the bones adjacent to the SI joints.

A 60-year-old man presents with pain in his hip. He denies other symptoms except for occasional night sweats. His calcium is elevated. Which of the following will have a normal bone scan? A) Avascular necrosis B) Osteoblastic type of malignancy C) Multiple myeloma D) Osteomyelitis E) Fracture

Answer C. Multiple myeloma Explanation Fractures, osteomyelitis, and osteoblastic type of malignancy will cause increased uptake on bone scan; avascular necrosis will cause decreased uptake. Multiple myeloma will have a normal bone scan because it causes lytic lesions, which don't enhance.

Your patient, a 52-year-old, 70-kg woman with a diabetic nephropathy and a stable creatinine of 2.5, falls and injures her knee. Her orthopedic surgeon orders an MRI of her knee with gadolinium to assess whether she has torn her anterior cruciate ligament or her medial meniscus. What is your appropriate response? A) Replace the MRI with a CT scan. B) Add knee radiographs to the existing MRI order. C) Perform the MRI without gadolinium. D) Perform a preoperative clearance assessment in anticipation of surgery. E) Replace the MRI with a bone scan.

Answer C. Perform the MRI without gadolinium. Explanation Nephrogenic fibrosing sclerosis (NFS) is a relatively new scleroderma mimic, which affects patients with renal dysfunction who receive gadolinium. The risk of developing NFS increases in patients with a creatinine clearance under 60 mL/min and increases tremendously in patients with a creatinine clearance under 30 mL/min. This patient has a creatinine clearance of 29.1; thus, she is at significant risk. As a result, unless the patient has a potentially life-threatening condition for which the gadolinium will improve the MRI's ability to provide critical data, this contrast agent should be avoided.

A 47-year-old woman presents with an acute painful ulcer on her left index finger. She has had classic triphasic color changes in her fingers for the past 18 months, precipitated by cold weather. She has noticed a 10-pound weight loss recently and is struggling to swallow solids. She has a 20-pack/year smoking history and has diet-controlled Type 2 diabetes mellitus. PHYSICAL EXAMINATION: BP = 137/87 Positive nailfold capillaroscopy with infarct at tip of index finger. Sclerodactyly noted. Multiple telangiectasias. All pulses intact. No bruits. Lung fields clear. Which of the following is not likely to be found? A) Anti-centromere antibody B) Rheumatoid factor C) Anti-SCL70 antibody D) Anti-histone antibody E) ANA

Answer D. Anti-histone antibody Explanation Anti-SCL70 is found in 8% of scleroderma patients with diffuse progressive disease. ANAs are frequently found in patients with scleroderma. Anti-centromere antibody is often found with CREST syndrome, which is limited cutaneous disease. Rheumatoid factors are nonspecific in this situation, but anti-histone antibodies are usually found in drug-induced lupus.

A 70-year-old female with RA presents with new-onset lower back pain. She has received methotrexate and prednisone for the past 20 years. She requests permission to increase her current prednisone maintenance dose from 5 mg qd to her "flare dose" of 60 mg qd. Exam reveals RA deformities but without acute synovitis. Her deep L-spine pain is not reproducible by palpation, and she has no pain with straight leg raises. No LE motor weakness or paresthesias. She denies groin pain. Which of the following is the most appropriate next step in management? A) MRI of both hips. B) Increase prednisone to 40 mg daily. C) Plain films of the spine. D) Start etanercept.

Answer C. Plain films of the spine. Explanation Rheumatoid arthritis does not, does not , affect the lumbar spine. Any patient with RA who presents with spine pain on the Boards has another cause for the pain. In this patient, chronic steroid use places her at risk for osteoporosis and compression fractures. Hence, she should receive plain films of her spine to look for them. Increasing prednisone or starting etanercept might be reasonable options if she had active rheumatoid disease, but the physical exam tells you that her joints show no active synovitis. An MRI of the L-spine or the hips might be necessary if the plain films do not suggest a cause of the pain. (It's possible the spine pain is referred hip pain from AVN.)

A 75-year-old woman presents to her geriatrician with pain in her shoulders and on the sides of her hips. She is being treated for a peripheral neuropathy with vitamin B12. She had a mastectomy 12 years ago, followed by chemotherapy, and has been told recently by her oncologist that she is disease-free. Examination reveals no evidence of synovitis, with excellent range of joint motion. No clear-cut proximal weakness. No tender points. No heliotrope and no rash. Normal funduscopic examination. Hb = 9.7 WBC = 4.9 ESR = 88 Calcium = 9.9 Chest x-ray normal Urinalysis negative Which of the following should she be treated with? A) Prednisone 60 mg/day B) Folic acid 1 mg C) Prednisone 15 mg/day D) Gabapentin E) Tricyclic antidepressant

Answer C. Prednisone 15 mg/day Explanation Her symptoms are typical of polymyalgia rheumatica: limb-girdle myalgia but no muscle weakness on physical exam, anemia, and elevated ESR. Low doses of prednisone (10 mg to 20 mg/day) are used for PMR alone while high doses of 60 mg/day of prednisone is for PMR with evidence of giant cell arteritis. Gabapentin would be used for her peripheral neuropathy. TCAs are used for nonarticular rheumatism such as fibromyalgia, which would not have an elevated ESR.

A 50-year-old male has a lengthy history of sinusitis. He has developed hemoptysis over the last week and feels a little short of breath. His energy levels have declined, and he has noticed increased night sweats. He has had increased hand, elbow, and knee pain over the last week. Physical examination reveals a cachectic male, blood pressure = 132/94. Crackles at both bases are noted. No synovitis. Erythematous papules in the lower extremities are present. Foot drop is present on left. Hb = 10.1 White cell count = 7.6 Creatinine = 1.7 Urine reveals trace protein with red cells and red-cell casts. C-ANCA = positive Later that same day, creatinine = 2.6 Renal biopsy is likely to show which of the following? A) Membranous nephropathy B) Focal sclerosis C) Rapidly progressive glomerulonephritis with crescents D) Interstitial nephritis E) Minimal change nephritis

Answer C. Rapidly progressive glomerulonephritis with crescents Explanation This patient clearly has granulomatosis with polyangiitis (formerly Wegener's) with the characteristic c-ANCA titer. With the proliferative sediment, trace proteinuria, and the rapid increase in creatinine, RPGN is the likely diagnosis. Minimal change usually is milder. Membranous nephropathy is associated with massive proteinuria (nephrotic syndrome). Focal sclerosis is usually seen in HIV and IV drug abusers.

Synovial biopsy is most helpful in diagnosing which of the following arthritic processes? A) Rheumatoid arthritis B) Postinfectious arthritis C) Tubercular arthritis D) Psoriatic arthritis

Answer C. Tubercular arthritis Explanation Inflammatory changes of synovial tissue are typically very nonspecific, and biopsy provides very limited information. An exception to this is septic arthritis due to tubercular organisms, for which synovial biopsy is the diagnostic procedure of choice.

A 19-year-old woman presents to your clinic with leukopenia, a positive ANA, arthritis, and a rash. You review her laboratory work and notice that her urinalysis shows significant proteinuria, red cells, and red cell casts. Her creatinine is 2.0 (normal < 1.3). Based on these findings, which antibody listed below should you test for at this time? A) c-ANCA B) SSA (Ro) antibody C) dsDNA antibody D) Anti-histone E) RNP antibody

Answer C. dsDNA antibody Explanation This patient likely has systemic lupus erythematosus. As active as her disease is, it would be good to check and see if she is dsDNA antibody-positive, which test is highly associated with lupus nephritis. The presence of a dsDNA would confirm the diagnosis of SLE in this patient. The anti-Sm antibody is specific for SLE. The RNP antibody is mainly seen with mixed connective tissue disease but can be found in SLE on occasion. The SSA (Ro) antibody is more helpful with neonatal lupus, Sjögren syndrome, and systemic sclerosis. Anti-histone is used to differentiate drug-induced lupus (but can be seen also in SLE). c-ANCA is seen with granulomatosis with polyangiitis (formerly Wegener's).

A 50-year-old female presents complaining of progressive weakness x 2 months. She requires extra time to get ready for work because her arms tire while she is combing her hair, and she is unable to climb a single flight of stairs without stopping for rest. She denies fever and pain or swelling of any joints or muscles. She is otherwise healthy. Exam is normal at rest. WBC: 9,500 (normal differential) Hgb: 12.7 Platelets: 325,000 Na: 145 K: 4.2 HCO3: 24 BUN: 12 Creat: 0.6 Glu: 92 CPK: 70 (50-170) Which of the following is the most appropriate next step in management? A) High-dose prednisone daily B) Peroneal nerve biopsy C) Anti-Jo antibodies D) Acetylcholine receptor antibodies E) Colonoscopy

Answer D. Acetylcholine receptor antibodies Explanation Differentiating rheum conditions from neurologic conditions can be a real challenge. Regardless of what section you're studying, get in the habit of constructing a problem list and looking for patterns/illness scripts. The problem list for this patient: Weakness (not pain!) with overhead movement Fatigue while going up stairs that improves with rest (not due to dyspnea) Exam is normal at rest. Normal CPK For what illness does this script fit? Myasthenia. The options ask you to choose between myasthenia gravis, a myositis (anti-Jo antibodies are associated with myositis; colonoscopy might be done to diagnose an occult malignancy associated with dermatomyositis, and peroneal nerve biopsies help diagnose confusing cases of myositis), and empiric treatment for polymyalgia rheumatica. Be prepared to see myasthenia presented on the exam and juxtaposed among these diseases in the differential diagnosis. This case is not PMR because the patient presents with weakness, not pain. Myositis should be considered, but the normal CPK excludes it ultimately. Myasthenia classically worsens with use and improves with rest. Acetylcholine receptor antibodies are present in most cases of myasthenia and can be used to diagnose the condition (replacing the Tensilon® test).

A 60-year-old man presents with complaints of a swollen left big toe for 3 days. This has never happened before. He has a negative past medical history. Examination shows a large swollen left big toe. This is likely to be acute gouty arthritis. Of the following agents, which would not be useful in the treatment of acute gouty arthritis? A) Oral steroid B) Intraarticular steroids C) Indomethacin D) Allopurinol E) Oral colchicine

Answer D. Allopurinol Explanation You would not use allopurinol for treatment of acute gouty arthritis! The nonsteroidal antiinflammatory drugs, oral colchicine (IV is no longer available in the U.S.), or oral or intraarticular steroids can be used in the acute presentation. Colchicine inhibits neutrophil activation by inhibiting crystal-induced protein tyrosine phosphorylation. Many patients, because of the GI side effects, do not tolerate oral colchicine. Allopurinol, remember, is for long-term management of gout; because it can prolong an attack, it should not be started for acute gout.

A 29-year-old psoriatic patient has noticed an increased stiffness of his lower back. He has a history of iritis. His psoriasis is controlled with local creams alone. His left knee was swollen 6 months ago but resolved spontaneously. Musculoskeletal exam reveals normal chest excursion, full ROM on flexion and extension of the spine, and tenderness at the sacroiliac joints. There is no peripheral soft tissue swelling of the joints. Which of the following is the next most appropriate investigation? A) Lumbar MRI B) Chest x-ray C) Slit-lamp investigation D) CT scan of SI joints E) HLA-B27

Answer D. CT scan of SI joints Explanation The patient has a seronegative spondyloarthropathy (probable ankylosing spondylitis) and is likely to be HLA-B27-positive, but this is very nonspecific and not diagnostic. CT scan of the SI joints is very useful for showing bony erosive changes at the sacroiliac joints. An MRI of the lumbar spine and chest x-ray would be indicated for late changes of AS on physical exam

A 59-year-old female presents to her primary care physician with a history of "finger problems." More specifically, her right index and left ring fingers have been "locking" and "triggering" for the past several months. She is beginning to lose function and is finding this most irritating. She is otherwise in excellent health. She has no joint pain particularly. About 18 months ago, she had a painful shoulder. She was seen by her local orthopedist and sent for a course of physical therapy, with fairly dramatic improvement of her shoulder pain and stiffness. Her diabetes is controlled on diet alone. A recent glycosylated hemoglobin (HbA1c) was normal. Past surgical history is positive for a hysterectomy and right carpal tunnel release 6 months ago. She has no other major symptoms of note at this time. Her physical examination reveals her to be a well-appearing, slightly obese female in no distress. Her blood pressure is 143/76 and pulse is 78. Cardiac and respiratory exam is benign. Musculoskeletal exam reveals evidence of asymmetric Heberden nodes. She has no active synovitis of the PIPs or MCPs. The wrists and elbows have a good range of motion. Shoulder abduction is full and normal. Lower extremity joints are normal. Her hands reveal evidence of sclerodactyly and slightly diminished grip strength. There is a positive prayer's sign. There is evidence of early Dupuytren contractures and slight thickening of some of her flexor tendons, including the two relevant fingers, which were not clearly demonstrated to trigger. Which of the following is the most likely diagnosis? A) Rheumatoid arthritis B) Diabetic autonomic neuropathy C) Progressive systemic sclerosis D )Diabetic-limited joint mobility syndrome

Answer D. Diabetic-limited joint mobility syndrome Explanation This patient has classic features of diabetic-limited joint mobility syndrome to include a positive prayer's sign, history of presumed "frozen shoulder syndrome," Dupuytren contractures, and carpal tunnel syndrome. She has pseudoscleroderma of Buschke. Other features include rotator cuff tendinitis, rotator cuff calcification, and bicipital tendinitis. If the stenosing tenosynovitis is caught early, which is responsible for her triggering, then a local corticosteroid injection is highly appropriate. Of course, there may be a temporary loss of diabetic control. However, if the stenosis of her tendon sheaths is fairly chronic, then a simple operative procedure by a hand surgeon would be appropriate.

Which of the following is not used for treatment of mild rheumatoid arthritis? A) Naproxen B) Prednisone C) Methotrexate D) Gold salts E) Cyclophosphamide

Answer D. Gold salts Explanation Gold salts are no longer used for RA. Previously, treatment of RA followed a pyramid regimen consisting initially of NSAIDs and/or low dose-glucocorticoids—with DMARDs added only as the disease got progressively worse. Now we know that RA-associated disability can be drastically reduced when we treat early disease aggressively with DMARDs. Current treatment paradigms focus on early diagnosis and early aggressive therapy to allow patients a chance at remission. The goal is to try to initiate DMARD therapy within 3 months of symptoms and if needed, to titrate drugs (add additional DMARDs or biologic agents) to attain low disease activity or remission.

See image.--> Osteophyte formation at DIP In this picture, what is the name given to the physical features identified by the arrows? A) Rheumatoid nodule B) Bouchard node C) Tophi D) Heberden node E) Ganglion cyst

Answer D. Heberden node Explanation Heberden nodes are due to osteophyte formation of the DIP joints. Bouchard nodes are the name given to osteophytes of the PIP joints. A ganglion cyst is a soft tissue bulge due to a synovial cyst that can form at any joint but is most often seen in wrists and knees. Occasionally, these cysts can occur over the DIP and PIP joints, but would be soft and fluid-filled, not bony as are Heberden nodes. Soft, somewhat rubbery, mobile rheumatoid nodules can develop in seropositive RA patients. These can appear over DIP joints but are not hard and bony. Tophi are a result of gout. These are soft, yellowish nodules that overlay joints and digits. Physical findings of OA include evidence of osteophytes, such as the Heberden and Bouchard nodes. There is also squaring of the base of the thumb and crepitus of joints, especially the knees. These findings, along with a history of brief morning stiffness in the presence of normal inflammatory markers, confirm the diagnosis of OA.

A 60-year-old Hispanic man comes in with recurrent episodes of palpable purpura with joint aches. PHYSICAL EXAMINATION: Significant for scattered purpuric papules on his lower extremities bilaterally Mild tenderness of the proximal interphalangeal joints of both hands and feet No synovial thickening is noted LABORATORY: Hemoglobin: 10.5 g/dL WBC: 7,000/µL MCV: 88 ESR: 40 mm/hr AST: 50 U/L Alkaline phosphatase: 90 U/L ANA: Negative Rheumatoid factor: 1:2080 (< 1:40 normal) Serum cryoglobulins: Markedly elevated Biopsy of skin: Leukocytoclastic vasculitis Which of the following tests will help you confirm your diagnosis? A) Serum and urine immunoelectrophoresis B) Antineutrophil cytoplasmic antibodies C) Anti-DNA D) Hepatitis C serology E) CT of chest

Answer D. Hepatitis C serology Explanation This patient has cryoglobulinemia. It is mixed due to the abnormally high rheumatoid factor binding to either monoclonal or polyclonal IgG. Vasculitis with purpura is the most common presentation for mixed cryoglobulinemia. Hepatitis C has a very strong association with mixed cryoglobulinemia! In one study, 86% of cryoglobulinemics were positive for hepatitis C! The monoclonal type is due to multiple myeloma or Waldenström macroglobulinemia—with this you would expect to see severe hemolytic anemia and something in the history like Raynaud's or cyanosis with exposure to cold.

A 34-year-old woman presents with a 3-month history of photosensitivity, intermittent rash on her face and cheeks, and diffuse myalgias. She has bilateral arthritis of the wrists. Her physical examination today is unremarkable except for the rash on her face/cheeks. Laboratory: ESR 35 mm/hr WBC 7,600/mm3 and a normal differential Hemoglobin 11.0 g/dL ANA is positive Anti-dsDNA is positive Anti SS-A is positive Anti-histone antibodies are positive C3 and C4 are low Which of the following is true regarding the laboratory findings and her history/physical examination? A) Elevated anti-U1RNP would suggest a worse prognosis in her. B) DR4 allele is associated with anti-Ro/SS-A. C) The positive anti-histone antibody indicates she has drug induced lupus. D) High levels of anti-dsDNA and low complement levels are associated with increased disease activity and development of nephritis. E) Anti-Sm is positive in these patients > 75% of the time.

Answer D. High levels of anti-dsDNA and low complement levels are associated with increased disease activity and development of nephritis. Explanation Her signs, symptoms, and laboratory data support the diagnosis of SLE. She has both a positive ANA and a positive anti-dsDNA, photosensitivity, anemia, and symmetric arthritis. A high titer of anti-dsDNA (as well as the low complement levels) is associated with increased disease activity and the development of nephritis. Anti-Sm antibody is found in only 17-30% of patients with SLE. Anti-histone antibody is found in nearly 100% of those with drug-induced lupus, but can also be seen in SLE that is not drug-induced. An elevated anti-U1 RNP actually suggests a better prognosis. DR3 allele is associated with anti-Ro/SS-A antibody, which is associated with a significant risk of neonatal lupus and can result in congenital heart block in the infant.

A 28-year-old man complains of a 3-month history of morning lower back pain, lasting 2 hours. He also noticed a swollen left second toe for 2 months, has had pain in the bottom of both heels when he walks, and has a scaly skin rash along his scalp hairline. What is the most appropriate initial therapy? A) Etanercept B) Celecoxib C) Glucosamine D) Methotrexate E) Prednisone

Answer D. Methotrexate Explanation The ideal treatment for a patient with psoriatic arthritis (PSA) such as this includes a non-biologic disease modifying anti-rheumatic drug (DMARD) (such as methotrexate) and a tumor necrosis factor (TNF) inhibitor (such as etanercept). However, prior to initiating a TNF inhibitor, most health insurers require an adequate and unsuccessful trial with a DMARD. No definitive data exists that glucosamine improves joint pain and is not considered an antiinflammatory agent. Celecoxib, a COX-2 specific inhibiting nonsteroidal antiinflammatory drug, is inadequate to suppress the inflammation in an inflammatory arthropathy (such as PSA), except the crystal arthropathies. Prednisone is not an appropriate therapy for this patient. Therefore, the answer is methotrexate.

A 19-year-old girl with a kidney transplant sees you for left groin and knee pain of 3 weeks duration. She originally had renal failure as a result of focal segmental glomerulosclerosis with nephrotic syndrome that was treated with high doses of steroids for nearly one year. She underwent a cadaveric kidney transplant 5 years ago, and her creatinine has been stable at 1.9. She is maintained on mycophenolate mofetil, tacrolimus, and prednisone. There is no history of trauma. On musculoskeletal exam, you find no joint abnormalities or limited range of motion, but she clearly has a limp on her left side when she walks. Your next step in evaluation of this patient would be? A) NSAIDs for control of pain and inflammation. B) Obtain a DEXA scan. C) Arthrocentesis of the left knee for possible occult infection. D) Obtain an MRI of the left hip. E) Kidney biopsy for occult rejection.

Answer D. Obtain an MRI of the left hip. Explanation Long-term and high-dose steroid use is associated with avascular necrosis of the hip, which may present as groin pain referring to the knee. The musculoskeletal exam can be normal in the early stages of avascular necrosis, with pain on weight-bearing as the only sign. There is no indication to tap the knee, although this patient is at increased risk for infection. Her creatinine is stable, and her knee pain would not be explained by rejection. A DEXA scan is likely to reveal diffuse lowering of bone mineral density, but not a diagnosis. NSAIDs might be useful symptomatically, but must be used with extreme caution in the setting of abnormal kidney function and won't help in the evaluation. An MRI of the hip is highly sensitive and specific in the detection of early stages of avascular necrosis of the hip.

A 70-year-old Caucasian man presents with a 3-month history of muscle pain, especially in his shoulders and pelvic girdle. He has severe morning stiffness on arising. On physical examination no muscle atrophy is noted. Upper extremities: No weakness noted Lower extremities: No weakness noted Laboratory: ESR is 120 mm/hr; ANA is 1:40 with a speckled pattern. Which of the following is the most likely diagnosis? A) SLE B) Rheumatoid arthritis C) Polymyositis D) Polymyalgia rheumatica E) Multiple myeloma

Answer D. Polymyalgia rheumatica Explanation This is an elderly man with shoulder and pelvic girdle muscle pain that has lasted longer than 1 month. It is more common in Caucasians than others. He has no muscle atrophy and no true weakness identified on examination. He reports severe morning stiffness and has an ESR of > 100 mm/hr. Treatment with 15 mg of prednisone will make this man feel much better quickly which is also considered a diagnostic response typical for PMR

A 50-year-old man presents with swelling in his left knee for 3 days. The knee is painful and he cannot ambulate well. He denies history of trauma to the knee. An aspirate of the knee is done and shows crystals with weakly positive birefringence on compensated polarized light microscopy. Based on the findings, which of the following is the most likely etiology? A) Gout B) Osteoarthritis C) Rheumatoid arthritis D) Pseudogout E) Infection with a crystal-producing organism

Answer D. Pseudogout Explanation Calcium pyrophosphate dihydrate (CPPD) crystals cause chondrocalcinosis (calcium in the cartilage). Pseudogout is the term used for the acute presentation of the disease. The crystals of pseudogout have weakly positive birefringence on compensated polarized light microscopy as opposed to gout where the crystals have strongly negative birefringence under polarized light.

A 50-year-old woman with complaints of joint pain for several months presents for evaluation. She reports that she has had morning stiffness for the last 3 months that lasts over an hour and gradually improves throughout the day. She has had swelling of her wrists and PIP joints for about 2 months and some swelling in her knees for about 6 weeks. Physical examination shows marked swelling of her wrists and PIP joints that is symmetrical. You note some nodules on the dorsum of her hands as well. Plain film x-rays of her hands show erosive synovitis. Which of the following is her most likely diagnosis? A) Psoriatic arthritis B) Pseudoxanthoma elasticum C) Ankylosing spondylitis D) Rheumatoid arthritis E) Osteoarthritis

Answer D. Rheumatoid arthritis Explanation Based on the older 1987 ACR criteria for RA, she has all of the classic features listed in her history and physical: Morning stiffness > 45 min for 6 weeks Swelling of wrists, PIPs for > 6 weeks Swelling of 3 joints for > 6 weeks Symmetric joint involvement Rheumatoid nodules Erosive synovitis x-ray changes of the hands (also positive RF factor not mentioned here). Realize that the new 2010 ACR/EULAR Criteria for RA is based on minimal joint involvement based on a point system: even 1 joint involvement (2 points), high levels of RF/ACPA (3 points), and high ESR/CRP (1 point) can be classified as RA on a few weeks ( < 6 weeks) of symptoms. These criteria have a provision whereby some patients can be classified as RA even if they do not fulfill the criteria designed to capture the undifferentiated or early rheumatoid patient before joint destruction occurs.

A 55-year-old Caucasian male works as a construction worker. He has a long history of tobacco use and presents to you with a complaint of low back pain that started about 3 weeks ago. He was referred to you by a pulmonologist in your practice. He has a history of asthma. He cannot remember what his medications are except for sildenafil. You suspect he may have osteoporosis even though he is a male. Which of the following medications could he be taking that would put him at risk for osteoporosis? A) Ranitidine B) Warfarin C) Sildenafil D) ACE inhibitors E) Chronic oral steroid use

Answer E. Chronic oral steroid use Explanation With his asthma history, you have to wonder if he is on chronic steroids. These will induce early osteoporosis, especially in men. The other drugs listed are not associated with early osteoporosis. Other risk factors for his osteoporosis can include hypogonadism.

A 64-year-old woman presents to her internist with swelling of her neck. She has had 5 years of dryness in her eyes and mouth. She has had pain in her knuckles. She is presently on celecoxib and hydroxychloroquine. Her hypertension is controlled with a thiazide. PHYSICAL EXAMINATION: Heberden nodes in both hands Mild MCP synovitis is noted bilaterally Significant caries are noted Crackles are noted at both bases Several rubbery 2 x 2-cm lymph nodes are noted in her left cervical chain White cell count = 2.9 ESR = 83 ANA = 1/1280 RF = neg. Uric acid = 4.9 Anti-SSA antibody = 1/32 Which of the following is the most likely diagnosis? A) Myelodysplastic syndrome B) AIDS with tuberculosis C) Pulmonary fibrosis D) Sjögren syndrome with non-Hodgkin lymphoma E) SLE with reactive lymphoid hyperplasia

Answer D. Sjögren syndrome with non-Hodgkin lymphoma Explanation With the sicca symptoms, +ANA, +SSA, the patient obviously has Sjögren syndrome. The lymph nodes sound ominous in the description, and there is a significantly increased risk of non-Hodgkin lymphoma in patients with Sjögren syndrome; this is the best answer for the question. In general, there is a higher risk of non-Hodgkin lymphoma with most of the connective tissue disease processes.

A 27-year-old female with seropositive erosive nodular RA currently receives methotrexate and infliximab (Remicade®) with good disease control. She calls your office because last night she saw a television show on the discovery channel about Jonas Salk and the history of vaccines. Which vaccine should she not receive? A) H1N1 influenza vaccine B) Neisseria meningitidis vaccine C) Tetanus toxoid D) Varicella vaccine E) Hepatitis B vaccine

Answer D. Varicella vaccine Explanation Patients who receive biologics (such as infliximab) and cytotoxics (including cyclophosphamide, azathioprine, and mycophenolate mofetil) should receive their immunizations as recommended by the CDC. However, because they are significantly immunosuppressed, they should only receive non-live virus vaccinations because they are at risk of dissemination of the virus and clinical illness. Therefore, protein, polysaccharide, and viral particle vaccines are acceptable. Tetanus toxoid is a purified and modified form of tetanus toxin. The H1N1 influenza vaccine is composed of many viral fragments, and the hepatitis B vaccine contains one of the hepatitis B outer envelope proteins. The Neisseria meningitidis vaccine contains polysaccharides from N. meningitidis. However, the varicella vaccine is made from live attenuated virus.

A 47-year-old man with benign prostatic hypertrophy develops prostatitis. He is placed on terazosin and ciprofloxacin. His other medications include omeprazole (reflux), erythromycin (acne rosacea), and lisinopril (hypertension). He returns 3 weeks later following a ski vacation with a severely painful left Achilles tendon. Of the following, what is the most likely explanation? A) Overuse due to recent ski trip B) Drug interaction due to lisinopril/ciprofloxacin C) Drug interaction due to erythromycin/terazosin D) Adverse effect due to omeprazole E) Adverse effect due to ciprofloxacin

Answer E. Adverse effect due to ciprofloxacin Explanation This patient develops Achilles tendon pain after a ski vacation. An acute injury from skiing is a possibility, but in thr5one-induced tendonitis/tendon injury is more likely. Tendon rupture related to quinolone use has been well documented in the literature, with the most cases attributed to ciprofloxacin. It is a side effect of the drug class and not related to any drug interactions.

A 37-year-old African-American female presents with symptoms of fatigue. She is ordinarily used to exercising 5-6 times a week and recently has not been enjoying her light weight-lifting and yoga exercises. She feels short of breath and tired during exercise and has noticed some aching in her finger joints and her knees. In general, she has been in excellent health. Her weight is constant and she denies any fevers, night sweats, or rashes. She has been waking up a little short of breath at night but blames this on some anxiety. She is presently going through a divorce. On exam she is alert and oriented, well-nourished, and well-hydrated. Height 5'6", weight 127, BP 112/76, P 64. Cardiac exam is benign and respiratory tract reveals some crackles at both bases. There are no rashes. As part of her workup, she has a normal TSH, creatinine of 0.6, SGOT of 127 with the upper limit of normal being 35, and SGPT of 126 with upper limit of normal being 36. CK is 17,300 with the upper limit of normal being 160. Which of the following autoantibodies is likely to provide the most significant insight into her disease process? A) C3 level B) Rheumatoid factor C) ANA D) Anti-U1-RNP E) Anti-Jo-1

Answer E. Anti-Jo-1 Explanation This auto-antibody is associated with pulmonary fibrosis in patients who have dermatomyositis or polymyositis. About 20% of patients with dermatomyositis and polymyositis have pulmonary fibrosis. In general, this auto-antibody is not associated with malignancy but is associated with a somewhat poorer prognosis, given the presence of the pulmonary involvement. The ANA and RF could possibly be positive but would not provide any further useful information. Anti-U1-RNP (in high titer) is very sensitive for mixed connective tissue disease and may be slightly interesting but not particularly in her case.C3 would be helpful if you were thinking lupus erythematosus. Many arthritic processes are clinically very similar in presentation. Synovial biopsy can help provide diagnostic information in certain circumstances.

A woman presents to the ED with her 3rd spontaneous abortion. Which of the following antibodies is associated with an increased risk of this occurring? A) c-ANCA B) Anti-Ro/SS-A C) Antihistone antibody D) Anti-La/SS-B E) Anticardiolipin antibody

Answer E. Anticardiolipin antibody Explanation The antiphospholipid syndrome (APS) is characterized by vascular thrombosis or pregnancy morbidity (typically miscarriages) and the presence of one or more autoantibodies. Autoantibodies associated with APS include: lupus anticoagulant, anticardiolipin, and B2 glycoprotein 1.

A 29-year-old female is seen in the outpatient setting for painful swollen knees and feet. These symptoms began about 10 days ago. She describes the pain as incapacitating. She can barely ambulate and is brought in today in a wheelchair. She has never had any joint pain prior to this. She denies any rashes. She thinks that she had a fever about 2 weeks ago, which developed 2-3 days into a week-long trip to Mexico. Toward the end of that trip, she developed fairly significant diarrhea for about 48 hours, which was quickly self-limiting. No rectal bleeding was noted at that time. She has had no urinary symptoms. On examination, her temperature was 97.3° F. She was not pale or jaundiced. BP was 112/72 and pulse was 100. Cardiac and respiratory exams were benign. Cutaneous exam was benign. Musculoskeletal exam revealed no upper extremity joint synovitis. However, she had very significant knee effusions, was very tender over the insertion of the right Achilles and had significant sausaging asymmetrically of a few of the lower extremity digits. Which of the following is the proper course of action at this time? A) Admit the patient and place her on broad-spectrum IV antibiotics. B) Inject both knees with corticosteroids. C) Give systemic corticosteroids. D) X-ray both knees and ankles. E) Check baseline labs to include CBC, sed rate, RF, and aspirate one of the knees.

Answer E. Check baseline labs to include CBC, sed rate, RF, and aspirate one of the knees. Explanation This patient has a classic reactive arthritis, probably acquired through a GI route. She provides no history to suggest any other reactive features to include urethritis or conjunctivitis. IV antibiotics would obviously have been inappropriate. It would be more appropriate to aspirate the joint and have baseline labs checked before proceeding to systemic corticosteroids or local joint injection.

A 45-year-old male presents with a 3-month history of a tingly, lower-extremity rash, predominant lower-extremity joint pain, and a history of recent cold-induced color change of the hands. CBC, LFTs, and U/A are normal. Hepatitis panel: HAV: + Anti-HBc IgM: - Anti-HBc IgG: - HBsAg: - Anti-HBs: + HCV: + Here is a picture of the rash: Petechiae/purpura Which of the following is the most likely diagnosis? A) Meningococcemia B) Immune thrombocytopenic purpura C) Wegener's D) Polyarteritis nodosa E) Cryoglobulinemia

Answer E. Cryoglobulinemia Explanation The patient in the question has a chronic presentation, not an acute one, which helps to exclude meningococcemia. He is also HCV-positive, and we know that HCV is associated with cryoglobulinemia and can present as petechiae and purpura. The remaining options do not fit with this clinical presentation. PAN presents with abdominal pain and is most often associated with HBV, not HCV. The patient's anti-HBs positivity reflects vaccination for HBV. ITP is often diagnosed when asymptomatic, based on a screening CBC, or patients present with mucosal bleeding or petechiae. ITP is not associated with Raynaud's, arthralgias, or HCV. This patient's CBC also shows normal platelets. Wegener's can have associated petechial and purpuric skin rashes (and don't be thrown off by that, because the Boards often include skin rashes as part of a Wegener's presentation). But this patient has no history of either upper respiratory or lung complaints to suggest Wegener's. Don't forget: The Boards are asking you to recognize patterns. If they give you one (such as HCV, Raynaud's, arthralgias, and petechiae/purpura), then pick the most likely cause of the pattern (cryoglobulins). Don't force the question to reach for a less-defined diagnosis.

A 40-year-old man is evaluated because of a 2-week history of fatigue. However, on further questioning, you learn that he has had dysphagia with severe generalized weakness. He notes dyspnea on exertion at 20 feet and has developed significant joint and muscle pains. Finally, with all of this, he says he developed an itchy widespread rash. PAST MEDICAL HISTORY: Negative SOCIAL HISTORY: Works as a used car salesman; married for 18 years with 2 children, ages 10 and 5 PHYSICAL EXAMINATION: VS: BP 120/70, P 100, RR 20, Temp 100.6° F HEENT: PERRLA, EOMI Throat: Slight redness noted, no exudates Neck: Supple Heart: RRR without murmurs, rubs, or gallops Lungs: CTA Abdomen: Bowel sounds are present; no hepatosplenomegaly Extremities: No cyanosis or clubbing Swelling and tenderness of the metacarpophalangeal and knee joints bilaterally Flat-surfaced, reddish-to-violet, scaling papules noted on the knuckles Motor exam: Weakness of the proximal arm and leg muscles Skin: Red papular eruption is noted on his face, chest, and back Scaling noted with cracking on the skin of the palmar surface of his fingers LABORATORY RESULTS: WBC: 15,000/µL; 50% polys, 40% lymphs ESR: 50 mm/hr AST: 170 U/L ALT: 190 U/L Serum aldolase: 40I U/mL (NL: 0.8-3.0) Rheumatoid Factor: 1:1280 in speckled pattern ANA: 1:1280 in speckled pattern Which of the following is the most likely diagnosis? A) Rheumatoid arthritis B) Systemic lupus erythematosus (SLE) C) Scleroderma D) Chronic fatigue syndrome E) Dermatomyositis

Answer E. Dermatomyositis Explanation His exam is consistent with dermatomyositis. Note that he has Gottron papules on his knuckles, which is the most specific finding for this disease. The elevated aldolase is helpful; so too is the proximal muscle weakness. Another classic finding (but which he doesn't have) is the "heliotrope" rash on the upper eyelids. Remember the ANA is nonspecific and can be seen in other autoimmune diseases as well as in healthy individuals. The patient does not fulfill criteria for scleroderma, rheumatoid arthritis, or SLE. Given the abnormal labs and exam, he has more than just chronic fatigue syndrome.

A 65-year-old woman is coming in for a routine physical examination. She is concerned about postmenopausal osteoporosis. She currently performs in the circus and is worried about bone density problems when she jumps off of the trapeze. Which of the following tests would be the best to detect and monitor osteoporosis? A) Quantitative CT scan B) Radiography C) Single-photon absorptiometry D) Dual-photon absorptiometry E) Dual-energy x-ray absorptiometry

Answer E. Dual-energy x-ray absorptiometry Explanation Commonly known as DXA, this is the quickest method. Precision of the DXA approaches 1-2%. The quantitative CT scan and dual photon absorptiometry take a lot of time. The CT scan has the drawback of giving a lot of radiation.

A 48-year-old man from Thailand comes to see you for an office visit. He states that he has recurrent gout attacks and tophi. He has never been treated and takes no meds. He denies any other medical problems. Musculoskeletal exam reveals oligoarticular tophi in an asymmetric pattern at the distal and proximal interphalangeal joints without any joint synovitis. Labs: WBC 10,000 Hgb 14.0 g/dL BUN 10 mg/dL Creatinine 0.9 mg/dL Glucose 90 mg/dL CO2 20 mmol/L Serum chloride 111 mmol/L Serum potassium 4.0 mEq/L Serum sodium 138 mEq/L Uric acid 9.8 mg/dL Tophus aspirate confirms monosodium urate crystals. You want to initiate allopurinol. Which would you consider doing to reduce the risk for severe allopurinol sensitivity in this patient? A) Initiate allopurinol no higher than 150 mg/day. B) Get an HLA-B*51 gene marker. C) Start prophylaxis with an antihistamine when initiating allopurinol. D) Get an HLA-B*27 gene marker. E) Get an HLA-B*5801 gene marker.

Answer E. Get an HLA-B*5801 gene marker. Explanation The 2012 ACR gout management guidelines indicate that prior to initiating allopurinol, consider HLA-B*5801 screening in selected patients, specifically those higher-risk sub-populations for severe allopurinol hypersensitivity reaction (e.g., Koreans with stage 3 or worse CKD; Han Chinese and Thai irrespective of renal function). The guidelines also note that initial allopurinol dose should not exceed 100 mg/day (and for those with CKD stage 4 or worse, no higher than 50 mg/day); allopurinol dose should be titrated every 2-5 weeks to achieve target uric acid goal < 6.0 mg/dL. HLA-B*51 gene marker is thought to predispose patients to Behçet syndrome. HLA-B*27 gene marker predisposes patients to spondyloarthropathies.

A 25-year-old woman presents to your office for evaluation of a rash and muscle weakness for the past 6 weeks. On examination, you find proximal muscle weakness of the neck flexors, arms, and legs. She has a slight rash around her eyes and a scaly pink rash over her knuckles. A biopsy of proximal muscle is likely to show which of the following? A) Malignant transformation B) Disappearance of muscle fibers C) Denervation changes D) Fatty infiltration of muscle fibers E) Immune complexes around muscle fibrils

Answer E. Immune complexes around muscle fibrils Explanation This patient has dermatomyositis. Muscle biopsy in a patient with dermatomyositis will show infiltration of muscle fibers with lymphocytes that is more intense around vessels. Immunohistochemical analysis will reveal immune complex deposition in the areas of inflammatory infiltrate. EMG will show primarily myopathic changes. PFTs in patients whose lungs are affected may show restrictive changes with decreased volumes and low DLCO. Ordering cancer screening studies is inappropriate in this patient because, overall, only about 15% of adults with dermatomyositis have a malignancy, and she is only 25 years old. However, the number of patients with cancer increases to ~ 30% in adults over 65 years of age. Urine is usually normal in dermatomyositis. Complete blood count and ESR are also usually normal; CK, aldolase, LDH, ALT, and/or AST are elevated in most patients. Inflammatory myositis causes proximal muscle weakness and, in dermatomyositis, the classic rash (heliotrope and Gottron papules) is virtually pathognomonic for this diagnosis. Malignancy association is not present in children or usually in young adults, but the risk increases with age. The investigation of inflammatory myositis may involve a series of laboratory and diagnostic testing including muscle biopsy, EMG, and sometimes MRI of muscles showing edema of proximal muscle groups.

A 75-year-old female presents with severe right shoulder pain preventing movement. She has a slightly bloody shoulder effusion with no crystals on polarizing microscopy. A radiograph shows calcific tendonitis and humeral head erosion. The likely diagnosis is which of the following? A) Crystal negative pseudogout B) Crystal negative gout C) Humerus fracture D) Rheumatoid arthritis E) Milwaukee shoulder

Answer E. Milwaukee shoulder Explanation The stem refers to a patient with the classic description of hydroxyapatite arthropathy (basic calcium phosphate crystal), which causes the Milwaukee shoulder and calcific tendonitis. This is also the salt that is responsible for tumoral calcinosis. To diagnose pseudogout (calcium pyrophosphate [CPPD] arthropathy), a patient must have either the CPPD crystal or chondrocalcinosis. To diagnose acute gout, a patient must have intracellular monosodium urate crystals. Most rheumatologists do not believe in "crystal negative" crystal arthropathy. If this patient had RA, she should have had symmetric arthritis involving her hands and wrists, and a humeral fracture should be evident on radiograph. We can only visualize the hydroxyapatite crystal with alizarin red staining or electron microscopy. Thus, crystals will be absent on polarizing microscopy.

A 28-year-old man develops swelling, pain, and tenderness in his left ankle and right knee. He had severe diarrhea after a picnic 1 month prior to the onset of his arthritis. In between his episodes of diarrhea and arthritis, he had also developed "pink eye," which was mild and lasted only 3 days. He was treated for "gonorrhea" 2 weeks ago but continues to have some clear penile discharge. He wonders if he needs more antibiotics because "the gonorrhea has never lasted this long before." Which of the following is the most likely diagnosis? A) Pseudogout B) Gout C) Resistant gonococcal arthritis D) Ankylosing spondylitis E) Reactive arthritis

Answer E. Reactive arthritis Explanation Reactive arthritis (formerly Reiter syndrome) classically presents with a triad of urethritis, conjunctivitis, and arthritis and is the best answer for this question. The arthritis usually involves the large joint of the lower extremities and is asymmetrical. Causes of reactive arthritis are genitourinary infections from Chlamydia or Ureaplasma and gastrointestinal infections due to Salmonella, Shigella, Yersinia, Klebsiella, and Campylobacter.

You are called by nursing staff during the late evening for complaints of a patient who was admitted for neurologic symptoms following a fall from a tree resulting in head trauma and a fractured right forearm. He has been doing well for the previous 36 hours with improving mental status and was prepared to be discharged the following morning. His arm was casted earlier today after having been splinted for the initial two days. The nurse informs you that the patient is complaining of significant pain in the right hand that has been increasing over the previous four hours. Which of the following would be the most appropriate next step? A) Ask orthopedics to evaluate before discharge. B) Institute IV antibiotics after obtaining blood cultures. C) Titrate IV morphine to control the pain. D) Request radiographs of the right hand. E) Remove the right forearm cast immediately.

Answer E. Remove the right forearm cast immediately. Explanation Bony fractures are commonly painful lesions, and this pain is normally improved with immobilization of the affected bone. Any increase in pain following a casting procedure must be promptly and aggressively addressed. Serious damage to nerves, perfusion, and tissue can occur with a defect in the casting situation. Any cast can be reapplied; so, if in doubt, remove the cast. There is no evidence of trauma to the hand or infection in the initial presentation and further radiology studies or antibiotics should not precede prompt release of cast restrictions. Simple narcotic masking would likewise be completely irresponsible. Casual referral to orthopedics places the patient at risk for irreversible damage, and unless the orthopedist is readily available, intervention should not be postponed unnecessarily.

A 47-year-old woman has had severe erosive, nodular, seropositive rheumatoid arthritis for 7 years. She has responded well to a combination of infliximab, methotrexate, and low-dose prednisone. She presents to the emergency department with a 2-day history of hemoptysis, fevers, and night sweats. Physical examination reveals mild synovitis of MCP's bilaterally and left knee effusion. Nodules are noted at her elbows and hands but no vasculitis. She has diminished breath sounds at the right apex. Abdomen is soft and without masses. LABORATORY: ESR: 66 Hb: 9.7 Chest x-ray: Cavity at the right apex revealed Which of the following investigations would be the most appropriate? A) Bronchoscopy B) ACE levels C) Rheumatoid factor D) High-resolution CT scan of the chest E) Sputum for AFB

Answer E. Sputum for AFB Explanation The anti-TNF drugs (infliximab) have been associated with a higher risk of tuberculosis. All patients beginning biologic therapy, especially anti-TNF drugs, must have a chest x-ray and PPD done prior to the commencement of these drugs. The other tests would not identify Mycobacterium tuberculosis.

A 62-year-old man with a long history of osteoarthritis of the hips and knees presents with new onset of severe pain in his right hip. The pain is so severe that he cannot roll over to sleep on his right side because of the discomfort. The pain is exacerbated by any movement of his right hip. Physical examination is significant only for eliciting severe pain with palpation of the lateral aspect of his right hip. The pain is especially exacerbated by abduction of the hip. He has no skin lesions or lymphadenopathy. Which of the following is the best treatment for this patient? A) Surgery B) Oral prednisone C) Vancomycin and ceftriaxone D) NSAIDs E) Steroid injection

Answer E. Steroid injection Explanation He has the classic symptoms of trochanteric bursitis. This results in pain over the lateral aspect of the upper thigh and is made worse by movement of the hip or direct palpation over the area. It is common in patients with pre-existing arthritis. Steroid injection will provide prompt relief.

Which of the following drugs would you avoid prescribing if a patient is currently on methotrexate? A) Ciprofloxacin B) Azithromycin C) Levafloxacin D) Cephalexin E) Trimethoprim/sulfamethoxazole (TMP/SMX)

Answer E. Trimethoprim/sulfamethoxazole (TMP/SMX) Explanation Avoid prescribing trimethoprim-sulfamethoxazole (TMP/SMX) in patients on methotrexate due to risk for methotrexate toxicity. TMX/SMX can reduce renal clearance of methotrexate as well as contribute to heighten anti-folate effects. Penicillins can also reduce the clearance of methotrexate, and patients should be closely monitored while on the antibiotic. While the other antibiotics listed can interact with methotrexate to some degree, they can and have been used safely with methotrexate.

A 23-year-old man with low back pain for the past 6 months presents for evaluation. The pain is most intense in the morning and seems to improve with exercise. On physical examination, he is tender on palpation of the sacroiliac joints. He has reduced lumbar lordosis. There is no evidence of peripheral arthritis. Which of the following is true? A) X-rays of the hands should be performed. B) You should order a 24-hour urine for protein and creatinine. C) A urine sample should be sent for culture and sensitivity. D) He needs treatment with prednisone. E) X-rays of the sacroiliac joints are indicated.

Answer E. X-rays of the sacroiliac joints are indicated. Explanation This man likely has ankylosing spondylitis based on his history and physical examination. The x-ray findings in patients with ankylosing spondylitis can include sacroiliitis, calcification of the longitudinal ligaments and syndesmophytes. Note: It would be rare to find an ankylosing spondylitis patient who has spondylitis without sacroiliitis! The diagnosis must be differentiated from inflammatory bowel disease or psoriasis, which can give you similar findings as far as the arthropathy and sacroiliitis.

A 50-year-old woman with systemic lupus erythematosus (SLE) for 20 years comes to the clinic with a complaint of sudden onset of left hip pain without antecedant trauma. The pain is much worse when she tries to walk or move the hip. She has been on chronic steroids for most of those 20 years. Her SLE has been under fairly good control for the past 5 years. PAST MEDICAL HISTORY: Last hospitalization was 5 years ago for lupus nephritis exacerbation Hospitalized before that for lupus cerebritis 10 years ago SOCIAL HISTORY: Lives with her boyfriend, a 20-year-old computer operator Doesn't smoke or drink FAMILY HISTORY: Mother with SLE died at age 45 Father aged 70 and healthy Sister aged 55 with SLE PHYSICAL EXAMINATION: VS: BP 120/70, P 100, RR 19, Temp 98.7° F HEENT: PERRLA, EOMI TMs clear Throat: Clear Neck: Supple Heart: RRR with II/VI systolic murmur (no change; heard in the past) Lungs: CTA Abdomen: Benign Skin: No rash Extremities: Left hip is painful with any type of movement Internal rotation is limited X-ray of left hip shows osteopenia Which of the following is the most likely diagnosis? A) Slipped epiphyseal head of the femur B) Avascular necrosis of the hip C) Acute osteomyelitis D) Fracture of the hip E) Chronic osteomyelitis

B. Avascular necrosis of the hip Explanation Up to 1/3 of SLE patients on chronic steroids will develop avascular necrosis of a hip. Best way to diagnose it is with an MRI. The acute onset makes you steer away from the osteomyelitis diagnose. A fracture should have been seen on plain film.

A 26-year-old man presents with an acutely swollen knee. His aspirate contains cloudy fluid with 42,000 WBC (95% PMNs) and rhomboid-shaped weakly positively birefringent intracellular crystals. The evaluation for an underlying cause of his condition should include which of the following? A) CBC, TSH, CRP B) Ferritin, glycosylated hemoglobin, urate level C) TSH, ferritin, calcium D) RF, ANA, dsDNA E) Folate level, vitamin B 12 , thiamine level

C. TSH, ferritin, calcium Explanation The crystal described is a calcium pyrophosphate crystal—the cause of pseudogout. This condition typically occurs in older individuals or in previously injured joints. If the patient or the joint does not meet either of these conditions, the most common underlying conditions include hypothyroidism, hemochromatosis, and hyperparathyroidism. The appropriate screening tests for these disorders are TSH, ferritin, and calcium.

A 40-year-old medical publishing company executive presents with a 2-month history of episodic left shoulder pain after his weekly bowling match. This resolves after 1-3 days but returns following each time he bowls. He has been taking acetaminophen and ibuprofen to relieve the pain. Otherwise, he is without complaints. PAST MEDICAL HISTORY: Negative SOCIAL HISTORY: No sexually transmitted disease history No alcohol or smoking No drug use Married with 3 kids PHYSICAL EXAMINATION: Left shoulder: Excellent muscle bulk with no point tenderness Passive abduction to 90° causing pain in the deltoid region With the upper arm held at 45° of abduction, resisted active abduction, and adduction reveals normal strength without pain Internal and external rotation is normal without pain Which of the following best describes his condition? A) Thoracic outlet syndrome B) Bursitis of the subacromial bursa C) Rotator cuff tear D) Impingement syndrome E) Osteoarthritis of the shoulder

D. Impingement syndrome Explanation This refers to compression of the subacromial bursa or regional tendons in the space between the acromion and the humeral head. The biceps tendon and all the rotator cuff tendons go through this area. The passive abduction to 90 degrees causing pain in the deltoid region is the test that nails this one. A rotator cuff tear would cause more pain during active abduction than passive abduction. Bursitis of the subacromial bursa would do the same thing. Osteoarthritis of the shoulder is very rare. A thoracic outlet syndrome would cause pain that extends from the base of the neck, over the top of the shoulder, and down the arm.


Conjuntos de estudio relacionados

Chapter 4 - The Internal Assessment

View Set

EMT Chapter 8 Lifting and Moving Quiz

View Set

Biology: Concepts and Connections, 7th Edition, Chapter 8

View Set

Cardiovascular changes in Pregnancy.

View Set